You are on page 1of 135

KDU

MCQ Review
Pediatrics final MBBS
#MMS 33

PEDIATRICS MCQ REVIEW

NOTE

There were doubts in answering some MCQs ,so make sure to do your own
analysis before answering .

GOOD LUCK!

PEDIATRICS MCQ REVIEW – LKSM 12 Page 1


#MMS 33

Contents
DEVELOPMENT ............................................................................................................................................. 6
Mile stones ................................................................................................................................................ 6
Developmental delay ................................................................................................................................ 8
EMERGANCIES .............................................................................................................................................. 9
Plant poisoning ......................................................................................................................................... 9
Drugs poisoning ........................................................................................................................................ 9
ALS........................................................................................................................................................... 11
Primary survey ........................................................................................................................................ 11
Shock ....................................................................................................................................................... 12
Snake bite................................................................................................................................................ 13
CHILD PROTECTION .................................................................................................................................... 14
Non accidental injury .............................................................................................................................. 14
Child abuse .............................................................................................................................................. 15
GENETICS .................................................................................................................................................... 16
Inheritance patterns ............................................................................................................................... 16
Downs syndrome .................................................................................................................................... 18
Turner’s syndrome .................................................................................................................................. 19
RHEUMATOLOGY........................................................................................................................................ 20
SLE ........................................................................................................................................................... 20
JIA ............................................................................................................................................................ 20
Limping.................................................................................................................................................... 22
Septic arthritis ......................................................................................................................................... 24
Dermatomyositis ..................................................................................................................................... 24
Rheumatic fever ...................................................................................................................................... 26
Rickets ..................................................................................................................................................... 27
Myopathy ................................................................................................................................................ 28
PERINATAL /NEONATAL MEDICINE ........................................................................................................... 29
Neonatal resuscitation ............................................................................................................................ 29
Congenital abnormalities ........................................................................................................................ 30
New born screening ................................................................................................................................ 35
Neonatal deaths ...................................................................................................................................... 36
Neonatal hypoglycemia .......................................................................................................................... 36
Term new born and preterm .................................................................................................................. 37
Neonatal jaundice ................................................................................................................................... 40

PEDIATRICS MCQ REVIEW – LKSM 12 Page 2


#MMS 33

Respiratory distress ................................................................................................................................ 41


Prenatal infections .................................................................................................................................. 43
Maternal health ...................................................................................................................................... 43
GROWTH AND PUBERTY ............................................................................................................................ 44
Growth .................................................................................................................................................... 44
Short stature ........................................................................................................................................... 44
Precocious puberty ................................................................................................................................. 46
NUTRITION.................................................................................................................................................. 48
Breast feeding ......................................................................................................................................... 48
Complementary feeding ......................................................................................................................... 49
Iron deficiency......................................................................................................................................... 50
Zn deficiency ........................................................................................................................................... 50
Vitamin deficiencies ................................................................................................................................ 51
Anthropometry measurements .............................................................................................................. 52
PEM ......................................................................................................................................................... 52
Obesity .................................................................................................................................................... 53
GIT AND LIVER ............................................................................................................................................ 55
IBD ........................................................................................................................................................... 55
CLCD ........................................................................................................................................................ 56
Gastroenteritis ........................................................................................................................................ 56
Constipation ............................................................................................................................................ 58
Lactose intolerance ................................................................................................................................. 58
Intussusception ....................................................................................................................................... 59
Intestinal obstruction.............................................................................................................................. 60
INFECTION/ IMMUNITY.............................................................................................................................. 61
Dengue .................................................................................................................................................... 61
Typhoid ................................................................................................................................................... 62
Rabies ...................................................................................................................................................... 64
Varicella................................................................................................................................................... 64
Measles ................................................................................................................................................... 65
Viral fever /investigations ....................................................................................................................... 66
Kawasaki disease..................................................................................................................................... 66
Infectious mononucleosis ....................................................................................................................... 67
Worm infections ..................................................................................................................................... 68
Abscess .................................................................................................................................................... 68

PEDIATRICS MCQ REVIEW – LKSM 12 Page 3


#MMS 33

Skin infections ......................................................................................................................................... 69


Anti-biotics .............................................................................................................................................. 69
Vaccines and eradication ........................................................................................................................ 69
Immunodeficiency .................................................................................................................................. 72
RESPIRATORY.............................................................................................................................................. 73
Stridor ..................................................................................................................................................... 73
Bronchiolitis ............................................................................................................................................ 74
Whooping cough ..................................................................................................................................... 75
Asthma .................................................................................................................................................... 76
Bronchiectasis ......................................................................................................................................... 78
Pneumonia .............................................................................................................................................. 79
Tonsillitis ................................................................................................................................................. 81
Bronchitis ................................................................................................................................................ 81
Pneumothorax ........................................................................................................................................ 82
CARDIAC...................................................................................................................................................... 83
Sudden cardiac death ............................................................................................................................. 83
Heart failure ............................................................................................................................................ 83
Arrhythmias ............................................................................................................................................ 84
Congenital heart defects ......................................................................................................................... 85
KUB AND GENITAL ...................................................................................................................................... 89
Scrotal problems ..................................................................................................................................... 89
UTI ........................................................................................................................................................... 89
PUV.......................................................................................................................................................... 92
VUR ......................................................................................................................................................... 92
Nephritic syndrome ................................................................................................................................ 93
Nephrotic syndrome ............................................................................................................................... 95
Electrolyte imbalance and urine changes ............................................................................................... 96
PSYCHIATRIC ............................................................................................................................................... 98
Nightmares.............................................................................................................................................. 98
Mental retardation ................................................................................................................................. 98
Nocturnal enuresis .................................................................................................................................. 99
ADHD/ASD and others .......................................................................................................................... 100
MALIGNAT ................................................................................................................................................ 102
Renal tumors ......................................................................................................................................... 102
Bone tumors.......................................................................................................................................... 103

PEDIATRICS MCQ REVIEW – LKSM 12 Page 4


#MMS 33

Brain tumor ........................................................................................................................................... 104


Hematological malignancies ................................................................................................................. 105
HEMATOLOGY .......................................................................................................................................... 109
Anemia .................................................................................................................................................. 109
Iron deficiency anemia.......................................................................................................................... 111
Thalassemia........................................................................................................................................... 112
Clotting disorders .................................................................................................................................. 112
DERMATALOGY ........................................................................................................................................ 114
ENDOCRINE............................................................................................................................................... 117
DKA........................................................................................................................................................ 117
DM......................................................................................................................................................... 117
NEUROLOGY ............................................................................................................................................. 120
Epilepsy ................................................................................................................................................. 120
Meningitis /encephalitis ....................................................................................................................... 122
Febrile convulsions................................................................................................................................ 124
Inflammatory polyneuropathy .............................................................................................................. 124
Headache .............................................................................................................................................. 125
CP .......................................................................................................................................................... 126
MG......................................................................................................................................................... 127
Intra cranial hypetension ...................................................................................................................... 128
Neural tube defects .............................................................................................................................. 129
Muscular dystrophies............................................................................................................................ 129
OTHERS ..................................................................................................................................................... 130
Pathognomic clinical feature ................................................................................................................ 130
Drugs ..................................................................................................................................................... 131
STATISTICS ................................................................................................................................................ 133

PEDIATRICS MCQ REVIEW – LKSM 12 Page 5


#MMS 33

DEVELOPMENT

Mile stones
15 months old child was presented with delayed walking.He is a bottom shuffler. He can
standindependently and walk with support but can not walk without support.Other
developmentalmilestones are normal. Best next step of management is, (INTAKE32)
a. Neuro imaging
b. Reassure and review at 18months
c. Muscle biopsy
d. CECT brain
e. Physiotherapy

T
A child with normal development can stand with support, walk around furniture and can grasp ricefrom
a pot. What would be the approximate developmental age? (INTAKE31)
A. 6 months
B. 8 months
C. 10 months
D. 12 months
E. 15 months
T

8 month old baby presented to the clinic, (INTAKE 29)


A. He can sit without support for 2 minutes
B. His weight is around 9-12 kg
C. He can grab things with pincer grasp
D. He promptly turn to the sound 1 feet lateral to him
E. He can reach to an object
T F T F T

Child of 8 months is expected to(INTAKE 30)


1. pull himself up by cot railing – 10months
2. pick up rice grain from plate - 10 months
3. monosyllabic babbling – 6 months
4. friendly to strangers -
5. crawl on all 4s – 8/9 months
F F T F T

PEDIATRICS MCQ REVIEW – LKSM 12 Page 6


#MMS 33

Gross motor (months) Vision and fine motor


Head Control 3 Reaches 4
Sits with support 4-5 Transfers and Palmar Grasp 6
Sits without support 6-8 Pincer Grasp 10
Crawls 8 Scribbles 12-14
Stands with support 9 Draws
Walks around Furniture 10 Line 2 yrs
Walks without support 12- 15 Circle 3 yrs
Cross 4 yrs
Square 4 1/2 yrs
Triangle 5 yrs
Builds Towers
3 1 1/2 yrs
6 2

Speech
Vocalizes 3 Social
Localizes 6-7 Smiles responsively 6 weeks
Mono syllabic sounds 6 Separation Anxiety 8 months
Babbling 7 Dry by day 2 yrs
Speaks 2-3 words 1 yr Dry by night 2 1/2 - 3 yrs
Understands commands 1 1/2 Symbolic Play 1 1/2 yrs
yrs Parallel Play 2 yrs
Speaks in phrases 2 yrs Interactive Play 3 yrs

A 13-month-old baby boy was referred for developmental assessment. He can stand without support
and walk a few steps. He does not have a pincer grasp. He can speak 3 meaningful words and tries to
feed himself with a spoon. What is the next most appropriate step? (COL2021)
a. Hearing assessment
b. Visual assessment
c. Assess CPK levels
d. Perform electromyogram
e. Perform thyroid function tests

For pincer graph he should visualize it


T

PEDIATRICS MCQ REVIEW – LKSM 12 Page 7


#MMS 33

Developmental delay
Regarding child development (COL2016)
a) Prematurity needs correction up to 2 years
b) Gross motor development occurs in cranio caudal direction
c) Development delay in more than 2 domains is known as global development delay
d) CHDR is an effective tool in assessing development
e) Development level of -1SD defines development delay for that domain

T T T T F

 Global developmental delay (also called early developmental impairment) implies delay in
acquisition of all skill fields (gross motor, vision and fine motor, hearing and speech, language
and cognition, social/emotional and behaviour) : illustrated
 Global developmental delay is a term often used to describe young children whose limitations
have not yet resulted in a formal diagnosis of intellectual disability; it is often inappropriately
used beyond the point when it is clear the child has intellectual disability, usually age 3 years.:
nelson
 Global developmental delay usually presents in the first 2 years of life
 Development level of -2SD defines development delay for that domain
 Prematurity needs correction up to 2 years , There is no rule, but we correction up to 2 years
 First head control last go for legs stability so cranio caudal

20 months old child presented with delay in speech. He is able to run, climbing stairs, feed himselfand
scribble with a pen. He is pointing to objects when he wants. He doesn’t say any word, only babbling
sounds. Affectionate but has temper tantrums. Most crucial step of management in this child is, (INTAKE31)
A. Hearing assessment.
B. Thyroid function test.
C. Psychiatric referral.
D. Speech and language assessment.
E. Educational therapist referral.
T

PEDIATRICS MCQ REVIEW – LKSM 12 Page 8


#MMS 33

EMERGANCIES

Plant poisoning
A 14 year old girl presented to the emergency care unit due to the ingestion of 5 Oleander seeds. She
was treated with gastric lavage and activated charcoal. GCS 15/15 ,BP 110/70, PR 84. What is the most
appropriate next step of management? (COL2021)
a. Administer intravenous adenosine
b. Start cardiac pacing
c. Arrange a psychiatric referral
d. Cardiac monitoring
e. Reassure and discharge

Plant clinical features Management


Yellow olender Sinus bradycardia /arrhythmias Cardiac monitor /atropine
Cerbera mangus Cramps
Datura stramonium Mad/blind/dry/hot/red Non stimulating environment
Pysostagmin
Strichnus nux vomica Opistatonus /AKI /dilated pupil Symptomatic MX
/painful convulsion
Gloriosa superba (colchicine) Watery diarrea /hypovolemic Shock management
hyponatremia /hair Hyponatremia mx
loss/respiratory failure

Drugs poisoning
Gastric lavage and activated charcoal is indicated in(INTAKE32)
a. 3 yr old child ingested with 100ml of PCM 01 hour ago
b. Iron 300mg after 2 hours
c. Kerosin ingestion after 1 hour
d. Caustic soda after 1 hour
e. antacid 100ml after 2 hours

F F F
Activated charcoal:
 high surface area leads to adsorption of many drugs
 can be effective in reducing absorption of toxic agent if administered within 1 h of ingestion
 ineffective for iron, hydrocarbons, and pesticides Gastric lavage and induced vomiting no
longer recommended

PEDIATRICS MCQ REVIEW – LKSM 12 Page 9


#MMS 33

Which one of the following is an absolute contraindication for gastric lavage? (INTAKE31)
A. Infancy
B. Corrosive poisoning
C. Kerosene poisoning
D. Unconscious patient
E. 2 hr. after poisoning
T

2 year old boy presented after 3 hours with a history of ingestion of 6 tablets of Paracetamol. Childlooks
well. What is the best option of management? (INTAKE 29)
A. Give activated charcoal
B. Perfume gastric lavage
C. Immediately send blood for Paracetamol levels
D. Calculate the toxic dose and manage accordingly
E. Monitor regularly with liver function test
T F F T F

Detoxification
 Activated charcoal (50g/200ml) water can give up to 1 hr
 Gastric lavage within 2 hours
Investigation
 Paracetamol level in 4 hr / Plasma levels in >15 hs not value
 Hepatic necrosis – elevated transaminase –after 24 hrs
Regular monitoring
 CBS 1rly

 ALT/PT, INR/RFT - 12 hrly


NAC – [given within 8 hours is more effective]
 Start immediately > toxic dose *
 Check the blood PCM level and start NAC
*Toxic dose – 150mg/kg

PEDIATRICS MCQ REVIEW – LKSM 12 Page 10


#MMS 33

ALS
A 6-year-old child was brought to the emergency unit after RTA. The child was talking with the mother
and he suddenly collapsed. Pulse was undetectable. ECG on the cardiac monitor shows sinus rhythm.
What is the immediate next step in the management? (COL2021)
a. Give 0.9% NaCl bolus
b. Administer IV adrenaline
c. DC cardioversion
d. Perform urgent CT scan
e. Stabilize the cervical spine

CPR is the best

Primary survey
A 5-year-old boy presented to the emergency department with fever and altered consciousness. On
examination, he is tachypnoeic, tachycardic and saturation is 90% while on 15L/min of Oxygen via
facemask. He only responds to pain and bilateral pupils are equally reactive to light. What is the most
appropriate next step in management? (COL2021)
a. Intubate and ventilate
b. Commence IV acyclovir
c. Infusion of 0.9% NaCl
d. Perform bag mask ventilation
e. IV Dexamethasone

AVPU score
 Alert GCS =15
 Voice GCS= 13
 Pain GCS = 8
 Unconscious GCS= 6

PEDIATRICS MCQ REVIEW – LKSM 12 Page 11


#MMS 33

Shock
Regarding shock in children? (COL2021)
A. Wide pulse pressure is seen in septic shock
B. Diastolic pressure is low in compensated hypovolemic shock
C. Core and peripheral temperature change of 4 Celsius or more means the patient has
compensated shock
D. 10mL/kg fluid bolus is given for compensated dengue shock
E. IV adrenaline is the drug of choice for anaphylactic shock management.

T T F

 Distributive shock – wide pulse pressure due to reduction of dystopic pressure


 Diastolic pressure is normal in compensated hypovolemic shock /systolic pressure is drop
 But in uncompensated state both are dropping

Commonest cause for childhood shock are, (INTAKE 29)


A. Septic shock in fluid restriction
B. Sudden circulatory failure
C. Inadequate tissue O2 supply

Rapid intravenous therapy is indicated in shock from gastroenteritis. However, it may be harmful in
head injury, malnutrition or diabetic ketoacidosis. It was also harmful in a trial of severe febrile illness
without shock in children in Africa (FEAST trial).

A 12 yr old boy who has been treated with amoxicillin for a sore throat was admitted to the emergency
treatment unit with a widespread urticarial rash. He was drowsy, tachycardia n had arapid thready
pulse. There were ronchi all over the lung field. What is the best immediate treatment? (INTAKE 30)
1. Iv hydrocortisone
2. Iv clorpheneremine
3. IM adrenalin
4. inhaled salbutamol
5. iv salbutamol
T

IV fluids are indicated in, (INTAKE 29)


A. Child is having vomiting
B. Severe dehydration
C. Lactose intolerance
D. Paralytic ileus
E. Diarrhea more than 10 ml/kg
F T F T

PEDIATRICS MCQ REVIEW – LKSM 12 Page 12


#MMS 33

Sever lactose intolerance – lactase free formula milk other than that BF continue

Diarrhea cause severe dehydration –


 IV /oral /IV+oral fluids should give
 Ringer lactate or normal saline
 100ml/kg within 3 hours

Some dehydration
 ORS 75 ml/kg in 4 hourly

No dehydration
Maintains + ongoing losses

Snake bite
Antevenoum is least useful in(INTAKE32)
a. Sri lankan krait
b. Indian krait
c. Cobra
d. Russel viper
e. Hump nosed viper

WOF snake bites not need polyvalent antivenom serum in sri lanka(INTAKE 30)
1. Russell's viper
2. cobra
3. Hump nosed viper
4. common krait
5. Indian krait
T

PEDIATRICS MCQ REVIEW – LKSM 12 Page 13


#MMS 33

CHILD PROTECTION

Non accidental injury


Following are more suggestive of non-accidental injuries than accidental injuries, (COL2021)
a. A 2-year-old girl with multiple bruises with different stages of healing in both lower limbs
b. An 8-month-old child with burn injury on feet
c. A 10-month-old baby with a spiral fracture of the femur
d. 12-month-old baby presenting unconscious with retinal haemorrhage
e. 18-month-old baby with fracture of the humours and sub periosteal bone formation

T T T F T
More suggest May be Not likely
# Any # in non-mobile Skull # in # in school age child with clear
Rib # young evidence
Multiple # without reason Long bone #
Multiple # in different ages in young but
mobile
Bruises Specific shape Trunk with Around shins of mobile child
Around neck vague history
Around wrist and ankle
Buttock <2 years
Burns Any burn in non-mobile Chest in mobile toddler
Specific shape (cigarette /iron) Splash marks with clear history
Glove and stocking burns
Bites Bruising in the shape of a bite thought A witnessed biting of one toddler by
unlikely to have been caused by a another
young child

DDs for non-accidental injuries,


A. Accidental inury.
B. ITP.
C. Osteogenesis imperfecta.
D. Munchausen by proxy.
E. Erythema multiforme.
T T T T F

 Impetigo – cigarette burn /Staphylococcal scalded skin syndrome


 Bruising – Mongolian blue spots /bleeding disorders
 # - osteogenesis imperfecta
 Subdural haematoma and retinal haemorrhages in an infant – consider non-accidental injury
caused by shaking or direct trauma.

PEDIATRICS MCQ REVIEW – LKSM 12 Page 14


#MMS 33

6 months old child presented to the hospital. Which of the following is most suggestive of non-
accidental injury(INTAKE 30)
1. Bruises of multiple stages
2. Retinal hemorrhage
3. Multiple epiphyseal fractures
4. Cigar burn on palm
5. Reflex anal dilatation
T

 Retinal hemorrhage happens in shaken baby and brain injury /trauma to head
 Reflex anal dilation is the reflexive dilation of the human anus to a diameter greater than two
centimeters in response to the parting of the buttocks or anal stimulation, such as brushing with
a medical instrument

Child abuse
The persons who are invited to the clinical meeting in a sexual abuse are(INTAKE 30)
1. JMO
2. Pediatrician
3. psychiatrist
4. Medical officer of health
5. police officer
T T T F T

Essential participants in multi disciplinary team for child abuse in Sri Lanka,, (INTAKE31)
A. JMO.
B. Police officer.
C. Paediatrition.
D. Probation child care officer.
E. MOH.
T T T T F

Child Psychiatrist

PEDIATRICS MCQ REVIEW – LKSM 12 Page 15


#MMS 33

GENETICS

Inheritance patterns
2-year-old child investigated for renal disease and mother is waiting for renal transplant. Maternal two
uncles also have renal problem. What are the inheritance patterns? (COL2016)
a) AD
b) AR
c) Mitochondrial
d) XR
e) XD

T T F F F
Autosomal recessive inheritance
• Affected individuals are usually homozygous for the abnormal gene; each unaffected parent will
be a heterozygous carrier.
• Two carrier parents have a 1 in 4 risk of having an affected child.
• Risk of these disorders varies between populations and is increased by consanguinity.
• Autosomal recessive disorders often affect metabolic pathways, whereas autosomal dominant
disorders often affect structural proteins

Autosomal dominant inheritance:


• Most common mode of Mendelian inheritance.
• Affected individual carries the abnormal gene on one of a pair of autosomes.
• There is 1 in 2 chance of inheriting the abnormal gene from affected parent, but there may be
variation in expression, nonpenetrance, no family history (new mutation, parental mosaicism,
non-paternity), or homozygosity (rare)

X-linked recessive inheritance:


• Males are affected; females can be carriers but are usually healthy or have mild disease.
• Family history may be negative – many arise from new mutations or gonadal mosaicism.
• Identifying female carriers is important to be able to provide genetic counselling.
• All the female offspring of affected males will be carriers, but none of the male offspring can
inherit the mutation.
• Half of the male offspring of a female carrier will be affected and half of the female offspring will
be carriers.

PEDIATRICS MCQ REVIEW – LKSM 12 Page 16


#MMS 33

Heterozygous females are usually asymptomatic in, (INTAKE 29)


A. Beta Thalassemia
B. Hemophilia A
C. Duchenne Muscular Dystrophy
D. Achondroplasia
E. Sickle cell anemia
T T T F T

Heterozygous females are asymptomatic in


1. Beta thalassemia – A/R
2. Sickle cell anemia – A/R
3. Hemophilia - X link recessive
4. Duchene muscular dystrophy - X link recessive
5. Achondroplasia – A/D
T T T T F

Heterozygous males are asymptomatic in autosomal recessive disorders only


Heterozygous females are asymptomatic in both autosomal recessive and X link recessive disorders

Autosomal Dominant
 Achondroplasia
 Osteogenesis Imperfecta
 Ehlers-Danlos Syndrome
 Tuberous Sclerosis
 Neurofibromatosis
Autosomal Recessive
 CAH
 Cystic Firbosis
 Galactosaemia
 Thalassaemia
 Sickle cell disease
X Linked Recessive
 Duchenne’s
 Becker’s
 G6PD
 Haemophilia
 Fragile X Syndrome
X Linked Dominant
 Vit. D resistant rickets

PEDIATRICS MCQ REVIEW – LKSM 12 Page 17


#MMS 33

Which of the following inheritance patterns are correctly matched? (COL2021)


a. Marfan syndrome - autosomal dominant
b. Sickle cell anaemia - autosomal recessive
c. Becker muscular dystrophy - autosomal recessive
d. Neurofibromatosis 1 - x linked recessive
e. Rett syndrome - x linked dominant

T T F F T

Becker muscular dystrophy – X link


Neurofibromatosis 1 - autosomal dominant
Rett syndrome
 It occurs almost exclusively in girls
 No cure
 Symptoms include impairments in language and coordination and repetitive movements.
Mimics Tourette syndrome

Regarding Autosomal dominant disorders(INTAKE32)


a. Each pregnancy has a 25% chance of being affected
b. New mutations are not commonly seen
c. Every generation is affected
d. Variable expression is a feature
e. Marfan syndrome is an example

F T T T T

Downs syndrome
Most significant risk factor in developing Downs’s syndrome is(INTAKE31)
A. Having a sibling with Downs syndrome
B. High maternal age
C. High paternal age
D. Consanguinity
E. High parity
T

PEDIATRICS MCQ REVIEW – LKSM 12 Page 18


#MMS 33

Turner’s syndrome
Turner’s syndrome(INTAKE 30)
1. cause infantile lymphedema
2. Severe mental retardation
3. obesity
4. subfertility
5. XXX Chromosomal compliment
T F T T F

Turners features (XO)


 Neck webbing
 Wide caring angle
 Wide space nipple
 Delayed puberty
 Obesity – Turner’s associated with hypothyroidism so can have obesity
 Lymphedema in neonatal period

15 Years old girl presents with 1ry amenorrhea. What are the symptoms suggestive of Turners, (INTAKE31)
A. Height is on 25th centile.
B. High FSH level.
C. Mild cognitive impairment.
D. Absent breast development.
E. Weak femoral pulses.

SS – height below 2nd centile/0.4th centile


 High FSH – hyperegonadism
 Has widely spaced nipples
 Breast can develop with estrogen therapy
 Week femoral pulse due to co arctation of aorta

Turner’s Syndrome is associated with, (INTAKE 29)


A. XXX
B. Obesity
C. Infertility
D. Neonatal lymphoedema
E. Severe mentally retardation

F T T F

PEDIATRICS MCQ REVIEW – LKSM 12 Page 19


#MMS 33

RHEUMATOLOGY

SLE
A 12-year-old girl was recently diagnosed with SLE. She has a faint malar rash, and mild arthritis of the
elbows, knees, and small joints of the hands. UFR -unremarkable. What is the most appropriate
treatment for her? (COL2021)
a. Azathioprine
b. Cyclophosphamide
c. Hydroxychloroquine
d. Methotrexate
e. Prednisolone

T
Mild SLE- (e.g. Cutaneous or joint involvement)
 UV avoidance
 Topical steroids
 Hydroxychloroquine

Severe SLE- Involvement of vital organs, systemic involvement, vasculitis


 Systemic corticosteroids- prednisolone 1mg/Kg/day, Methyl prednisolone
 Steroid sparing drugs (Cyclophosphamide, azathioprine, methotrexate, cyclosporine,
mycophenalate mofitil)
 IV Immunoglobulins

JIA
10 year old girl who was diagnosed to have JIA has morning stiffness even though she is onhigh dose of
Ibuprofen. What is the next best step in management? Convert to(INTAKE31)
A. Naproxen Na
B. Intra articular steroid
C. Oral steroid
D. Low dose MTX
E. Infra red therapy
T

Medical management
 NSAIDs - Only relive symptoms during acute flares
 Intraarticular steroids - usually in monoarticular
 Systemic steroids - systemic JIA and macrophage activation syndrome

PEDIATRICS MCQ REVIEW – LKSM 12 Page 20


#MMS 33

 Methotrexate - Reduce joint damage


 Cytokine Modulators( Biologics)

About JIA(INTAKE32)

a. female preponderance in pausing


b. RF + indicate worse prognosis
c. polyarticular type only cause permanent disability
d. Systemic type has low grade fever

F T F T

 Polyarthritis – low grade fever


 Good prognosis – oligo articular persistent
 RF – moderate prognosis
 If untreated can have permeant disability

A 14-year-old girl presented with arthritis. Which of the following are more suggestive in the diagnosis
of SLE than polyarticular JIA? (COL2021)
A. Weight loss
B. Vasculitic rash
C. Oral ulcer
D. Alopecia
E. Thrombocytosis

F T T T F
SLE - AIHA /↓WCC /↓Plats

Polyarthritis (RF negative)


 Symmetrical large and small joint
 Marked finger involvement
 Cervical and temporomandibular joint involvement
 Low grade fever
 Uveitis
 Late reduction of growth rate

Polyarthritis (RF positive)


× Symmetrical large and small joint
× Marked finger involvement
× Like adult rheumatoid arthritis - Rheumatoid nodule

WOTF are true regarding JIA(INTAKE 30)


1. systemic jia accounts for 50% of cases
2. anterior uveitis is common in polyarticular type

PEDIATRICS MCQ REVIEW – LKSM 12 Page 21


#MMS 33

3. pericarditis is common
4. methotrexate is used as a disease modifying drug
5. intraarticular steroid injections are not recommended
T

 oligo articular 57 %
 polyarticular 19%
 Systemic 9%

 Anterior uvitis is commonest in oligo articular - so screen

 Common complications – Chronic anterior uveitis , Flexion contractures of the joints, Growth
failure, Anaemia of chronic disease, delayed puberty , Osteoporosis

 Intra articular injection – first line in oligo articular JIA (can give mono articular and polyaricular
JIA )

Limping
6 yr old boy presented with acute onset limp and shorting of leg. What is the most likely
diagnosis(INTAKE32)
a. TB hip
b. transient synovitis
c. perthes disease
d. poliomyelitis

T
Septic arthritis/ Osteomyelitis
 Febrile, toxic-looking infant, irritability with nappy changing
 Restricted joint range (especially hip) or limb movement

Transient synovitis of the hip / Perthes disease


 Sudden limp in an otherwise well young child
 Unilateral restricted hip movement

‘Growing pains’ Osteoid osteoma/ Leukaemia, lymphoma, neuroblastoma (young child)


 Nocturnal wakening with leg pain
 Anaemia, bruising, irritability, infections

A 6-year-old girl was presented with limping of the right lower limb for 2 days. She has a history of an
upper respiratory tract infection one week ago. On examination, she is apyretic and is found to have
restricted painful movements of the right hip joint.What is the most likely diagnosis? (COL2021)
a. Acute rheumatic fever
b. JIA
c. Septic arthritis

PEDIATRICS MCQ REVIEW – LKSM 12 Page 22


#MMS 33

d. Slipped upper femoral epiphysis


e. Transient synovitis

A 7 year old boy presented with limping in the left lower limb for 2 weeks following a fall. He complains
of pain with passive movement in L/thigh. Abduction and internal rotation is impaired. Whatcould be the
diagnosis? (INTAKE31)
A. Green stick fracture of femur
B. Subluxation of the epiphysis of femur
C. Septic arthritis of the knee
D. Perthes disease
E. Transient sinuvitis
T

1-3 year 3-10 year 10-15 years


Septic arthritis or Osteomyelitis Transient synovitis (Irritable Slipped Upper Femoral
hip) epiphyses(SUFE)
Developmental dysplasia of
hip(usually does not present Septic arthritis or osteomyelitis Septic arthritis or Osteomyelitis
with pain)
Perthes’ disease Perthes’ disease
Fracture or soft tissue injury
(toddler fractures or non Fracture or soft tissue injury Fracture or soft tissue injury
accidental injury) (Stress Fractures)

Transient synovitis Septic arthritis Perthes Disease


looks well Look ill Temporary interruption of the
blood supply to the proximal
hip movements - Comfortable Hip movements - Hip held femoral epiphysis, leading to
at rest, limited internal rotation flexed; severe pain at rest and osteonecrosis and femoral head
and pain on movement worse on any attempt to move deformity.
joint
have had a nonspecific Pain is activity related more in
upper respiratory tract infection evening
Rest, analgesia Joint aspiration Containment
 Casts/Orthoses
 Surgical

Limited weight bearing


Pain relief if necessary

PEDIATRICS MCQ REVIEW – LKSM 12 Page 23


#MMS 33

Septic arthritis
20 months old baby presents with fever and irritability for 3 days with a warm, swollen, tender R/Sknee
joint. What is the most important investigation in management? (INTAKE31)
A. FBC
B. CRP
C. Blood culture
D. Xray R/knee joint
E. USS of knee joint
T

20-month-old boy presented with fever and irritability for 3 days. He is unable to move his R/leg. O/E
there is R/knee joint swelling and tenderness. What is the most appropriate investigation before
management? (COL2016)
a) Blood culture and ABST.
b) FBC.
c) ESR.
d) USS knee joint.
e) X – Ray knee joint

Best is aspirate and culture

Dermatomyositis
12 year old girl presented with difficulty climbing stairs for 1 month. Was well before. Also had a
purplish rash on her eyelids’. Neurological exam revealed grade 4 power on all 4 limbs with normal
reflexes. Investigations are as follows:
Hb 12,
wbc 10100 (N-58%, L-36%),
platelets 220 000,
ESR 110
What is the most likely diagnosis? (INTAKE32)
a. Myasthenia gravis
b. Dermatomyositis
c. GBS
d. Limb girdle muscular dystrophy
e. Becker’s muscular dystrophy

PEDIATRICS MCQ REVIEW – LKSM 12 Page 24


#MMS 33

Dermatomyositis
Lethargy, unwilling to do physical activities, irritability, rash
Eyelid erythema Proximal muscle weakness

Limb girdle muscular dystrophy


These conditions present with proximal upper and lower limb weakness. Cardiomyopathy and difficulty
with breathing may be associated with some. These conditions can have different modes of inheritance.
Plasma creatine kinase is usually raised.

Becker’s muscular dystrophy


Proximal muscle weakness
Average age 11

Myasthenia gravis
↑ing muscular fatigue
 Extra-ocular: bilateral ptosis, diplopia
 Bulbar: voice deteriorates on counting to 50
 Face: myasthenic snarl on attempting to smile
 Neck: head droop
 Limb: asymmetric, prox. weakness
 Normal tendon reflexes
 Weakness worsened by pregnancy, infection, emotion, drugs (β-B, gent, opiates, tetracyclines)

GBS
 Symmetrical, ascending flaccid weakness / paralysis
 LMN signs: areflexia, fasciculations may occur
 Proximal > distal (trunk, respiratory, CNs [esp. 7])
 Progressive phase lasts ≤ 4wks

PEDIATRICS MCQ REVIEW – LKSM 12 Page 25


#MMS 33

Rheumatic fever
Regarding acute rheumatic fever, (COL2021)
A. Arthritis in large joints is a clinical manifestation
B. Chorea is a minor criterion
C. Neutrophil leukocytosis is seen
D. Phenoxymethylpenicillin is the treatment of choice
E. Corticosteroids are contraindicated

T F F F

Major criteria
 Arteritis
 Carditis
 Chorea
 Subcutaneous nodule
 Erythema marginatum

Minor criteria
 Fever
 Polyartheralgia (can’t take with polyarthritis)
 Raised CRP/ESR (>30 in high risk and >60 in low risk)/leucocytosis ?
 Prolong PR interval

TX
 Arthritis with severe carditis and pending heart failure - Steroids for 2-3 weeks
 Arteritis - Bed rest , Salicylates 100mg/kg (high doses) divided dose ± antacid
 Eradication of the group A strep - Single dose IM benzathine penicillin G 600,000 - 1,200,000
units (painful injection ) or Oral penicillin for 10 days
o Phenoxymethyl penicillin – penicillin V - Oral penicillin
o Benzathile penicillin – penicillin G – IM

12-year-old girl with past history of rheumatic heart disease presented with fever, right knee joint pain
and swelling for 3 days. Pan-systolic murmur and early diastolic murmur were detected. HR 120 bpm,
ESR 112, and WBC 20000 (N50%). 3 pus cells and 2 red cells in the UFR. ASOT was 400 and ECHO is
pending. (COL2016)
a) IE
b) Reactivation of rheumatic fever
c) JIA
d) Post streptococcal arthritis
e) SLE

PEDIATRICS MCQ REVIEW – LKSM 12 Page 26


#MMS 33

T
Early diastolic – AR
• Pulse – sinus rhythm ,large volume ,collapsing
• BP – wide pulse pressure
• Apex – displaced ,diffuse ,forceful
• Murmurs
o high pitched ,early diastolic at LSE
o Ejection systolic at base and into neck (high flow )
o Mid diastolic rumble at apex

Rhumatic fever criteria fulfilled


Jones criteria
 2 major + essential criteria *
 One major + 2 minor + essential criterion *

*markedly raised or rising ASO titer / positive rapid streptococcal antigen test /positive group A
streptococcus culture) [Anti streptokinase , Anti DNAase B , Anti hyaluronidase]

Rickets
4-year-old boy with bow legs. X rays show features of rickets. Calcium level - 2.05, Phosphate -2.42,
alkaline phosphatase – 1850. Next step of management? (COL2016)
a) do USS abdomen
b) Treat as nutritional rickets.
c) Arrange an orthopaedic referral.
d) Do a blood gas analysis.
e) Request serum creatinine levels.

• Calcium (serum) 8.6-10.3 mg/dL


• Calcium (ionized) 4.4-5.2 mg/dL
• phosphorous is 2.5-4.5 mg/dL

Rickets features –
 Delayed closure of fontanels
 frontal bossing
 dental hypoplasia
 pectus carinatum
 swelling in wrist
 wide sutures
 caraniotabes

PEDIATRICS MCQ REVIEW – LKSM 12 Page 27


#MMS 33

 rachitic rosary
 Harrison sulcus
 bowing of legs

Myopathy
6 years old child presented complaining refusing to walk. He had fever for 4 days and resolved.There was
severe calf tenderness and there was no other physical signs(INTAKE 30)
1. paralytic poliomyelitis
2. viral myositis
3. congenital myopathy
4. acute phase of polyarthritis nodusum
5. Hysterical convulsive syndrome

Polyarthritis nodosm – arthritis in large joints /bowel systems /fever

Viral myositis – muscle inflammation followed by a viral infection

Paralytic poliomyelitis – mild fever /headach – then recover and have weekness spred from one muscle
gp and spred up to respiratory system and can die

PEDIATRICS MCQ REVIEW – LKSM 12 Page 28


#MMS 33

PERINATAL /NEONATAL MEDICINE

Neonatal resuscitation
20 hrs. old child presented with breathing difficulties, intercostal recessions and with respiratory
distress. Heart sound in R/S and scaphoid abdomen noted. What is the immediate nextstep in
management? (INTAKE31)
A. Intubation
B. Oxygen therapy
C. Suck out
D. Oxygen via face mask

What is the incorrect step in neonatal resuscitation? (INTAKE 29)


A. IV Adrenalin
B. Ambu bag with 35 mmHg
C. 100% O2
D. Nasal airway
E. Low pressure suction

After severe birth asphyxia WOF are immediate complications(INTAKE32)


a. seizure
b. Hypercalcemia
c. Hypoglycemia
d. Hypotonia
e. Metabolic acidosis

Neonatal resuscitation, Chest expansion achieved after giving inflation breaths. Next step(INTAKE32)
a. Adrenaline IV
b. Ventilation breaths
c. Start cardiac massage
d. Intubate
e. Inflation breath continued

PEDIATRICS MCQ REVIEW – LKSM 12 Page 29


#MMS 33

Congenital abnormalities
Pediatric surgical emergencies which need immediate correction are, (INTAKE 29)
A. Inguinal hernia
B. Trachea esophageal fistula
C. Cephalohematoma
D. Cleft palate
E. Diaphragmatic hernia
F T F F T
 Inguinal hernia usually before discharge or can wait for 5 months
 Umbilical hernias wait for 3/2 years

 Cleft palate 6 months


 Cleft lip 3 months

 Trachea esophageal fistula –kinking of the NG tube in XRAY

Gold standard investigation for congenital pyloric stenosis(INTAKE32)


a. USS
b. Endoscopy
c. Barium meal
d. Golf sign in feeding test
e. Plain Xray

You are called to see an 8 hours old baby with features of Downs’s syndrome due to recurrent non-
bilious vomiting. The baby does not tolerate oral feeds. The baby is hemodynamically stable. What is
the most appropriate next step of management? (COL2021)
a. Urgent surgical referral
b. Perform an urgent USS of the abdomen
c. Do urgent x-ray
d. Keep Nil by mouth and commence IV fluids
e. Perform a sepsis screen and commence antibiotics

T
Pyloric stenosis
 More common in boys and those with a maternal family history
 Signs are: visible gastric peristalsis, palpable abdominal mass(olive) on test feed and possible
dehydration
 Associated with hyponatraemia, hypokalaemia and hypochloraemic alkalosis
 Diagnosis may be confirmed by ultrasound
 Treated by surgery after rehydration and correction of electrolyte imbalance.

PEDIATRICS MCQ REVIEW – LKSM 12 Page 30


#MMS 33

Regarding tracheo-oesophageal fistula, (COL2021)


A. H type is the commonest
B. Presents with respiratory distress at birth
C. Associated with recurrent pneumonia
D. Warrants immediate surgery
E. Associated with anal atresia

F T T T T

 Clinical presentation is with persistent salivation and drooling from the mouth after birth.
 Continuous suction is applied to a tube passed into the oesophageal pouch to reduce aspiration
of saliva and secretions pending transfer to a neonatal surgical unit.
 VACTERL association (Vertebral, Anorectal, Cardiac, Tracheoesophageal, Renal and Radial Limb
anomalies)

Regarding duodenal atresia (COL2016)


a) Bile stained vomitus is a feature
b) Causes hypokalaemia as a complication
c) Abdominal x-ray shows multiple air-fluid levels
d) Presents within 48 hours of life
e) Usually resolves spontaneously

T T T F
Duodenal atresia
• One-third have Down syndrome
• it is also associated with other congenital malformations
• The hallmark of duodenal obstruction is bilious vomiting without abdominal distention, which
is usually noted on the 1st day of life
• Abdominal X-ray in duodenal atresia showing a ‘double bubble’ from distension of the stomach
and duodenal cap. There is absence of air distally.
• Atresia or stenosis of the bowel and malrotation are treated surgically, after correction of fluid
and electrolyte depletion

**A meconium plug will usually pass spontaneously

Causes for vomiting bile in 1st week of life, (INTAKE31)


A. Duodenal atresia.
B. Neonatal hepatitis.
C. Normal neonate.
D. Necrotizing enterocolitis.
E. Congenital pyloric stenosis.
T F F T F

PEDIATRICS MCQ REVIEW – LKSM 12 Page 31


#MMS 33

Duodenal atresia
 Atresia or stenosis of the duodenum – one-third have Down syndrome and it is also associated
with other congenital malformations
 small bowel obstruction presents with persistent vomiting, which is bile-stained unless the
obstruction is above the ampulla of Vater.
 Meconium may initially be passed, but subsequently its passage is usually delayed or absent.
 Abdominal distension becomes increasingly prominent the more distal the bowel obstruction.
 High lesions will present soon after birth, but lower obstruction may not present for some days.
 Double bubble sign in x-ray abdomen

Neonatal hepatitis -
Necrotizing enterocolitis – presentation with bile vomiting in first few weeks of the life

Congenital displacement of hip joint is common in, (INTAKE 29)


A. Breech deliveries
B. Antenatal use of Sodium Valproate
C. First born child
D. Child of a diabetic mother
T F T F

Risk factors for CDHJ


 Family history
 Being girl
 Breech presentation
 Neuromuscular problems in baby
 Oligohydramnios
st
1 bone child

Not a recognized intervention of 1ry prevention of congenital anomalies(INTAKE31)


A. Introduction of Rubella vaccine
B. Control of diabetes mellitus during antenatal period
C. Avoid alcohol in pregnancy
D. Anomaly screening by USS
E. Folic acid supplementary
T

Match the disease with correct presentation (COL2016)


a) Bilateral choanal atresia - cyanosis while feeding
b) adrenal hypertrophy - recurrent ear infections
c) tracheoesophageal fistula - frothing

T F T
choanal atresia
 Clinical Manifestations New-born infants have a variable ability to breathe through their
mouths, so nasal obstruction does not produce the same symptoms in every infant.

PEDIATRICS MCQ REVIEW – LKSM 12 Page 32


#MMS 33

 When the obstruction is unilateral, the infant may be asymptomatic for a prolonged period,
often until the 1st respiratory infection, when unilateral nasal discharge or persistent nasal
obstruction can suggest the diagnosis.
 Infants with bilateral choanal atresia who have difficulty with mouth breathing make vigorous
attempts to inspire, often suck in their lips, and develop cyanosis.
 Distressed children then cry (which relieves the cyanosis) and become calmer, with normal skin
color, only to repeat the cycle after closing their mouths.
 Those who are able to breathe through their mouths at once experience difficulty when sucking
and swallowing, becoming cyanotic when they attempt to feed.

Indications for the removal of both the tonsils and adenoids are controversial but include:
 recurrent otitis media with effusion with hearing loss, where it gives a significant long-term
additional benefit
 obstructive sleep apnoea (an absolute indication )

6 weeks old baby presented with inspiratory stridor. Mother said that it was present soon after thebirth
and lasted for few weeks and was gradually increasing. Otherwise the child is normal. The stridor gets
worsened in supine position. What may be the most possible diagnosis? (INTAKE31)
A. Laryngeal webs
B. Paralysis of vocal cords
C. Laryngeal stridor
D. Vascular ring around trachea
E. Choanal atresia
T

Choanal atresia
 This is the most common congenital anomaly of the nose
 Association with CHARGE syndrome
 Obstruction is unilateral; the infant may be asymptomatic for a prolonged period, often until the
1st respiratory infection
 Bilateral - have difficulty with mouth breathing make vigorous attempts to inspire, often suck in
their lips, and develop cyanosis

Laryngeal webs
 Airway obstruction is not always present and may be related to the subglottic stenosis.

Paralysis of vocal cords


 Congenital central nervous system lesions such as myelomeningocele, Arnold-Chiari
malformation, and hydrocephalus may be associated with bilateral paralysis
 high-pitched inspiratory stridor

Vascular ring
 The term vascular ring is used to describe vascular anomalies that result from abnormal
development of the aortic arch complex. The double aortic arch is the most common complete
vascular ring, encircling both the trachea and esophagus, compressing both.

PEDIATRICS MCQ REVIEW – LKSM 12 Page 33


#MMS 33

 With few exceptions, these patients are symptomatic by 3 mo of age. Respiratory symptoms
predominate, but dysphagia may be present.
 Diagnosed by barium swallow
Laryngomalacia
 Laryngomalacia is the most common congenital laryngeal anomaly and the most common cause
of stridor in infants and children
 Stridor is inspiratory, low-pitched, and exacerbated by any exertion: crying, agitation, or feeding.
 Symptoms usually appear within the first 2 wk of life and increase in severity for up to 6 mo,

Regarding cleft palate, (INTAKE31)


A. Associated with trisomy 13
B. If corrected within 6 months gives best cosmetic results.
C. Breast feeding contraindicated.
D. Associated with otitis media.
E. Associated with alcohol intake in pregnancy.
T T F T T

Some are associated with maternal anticonvulsant therapy and chromosomal abnormalities (Patau
syndrome (trisomy 13)
They may be detected on antenatal ultrasound scanning.

Patau syndrome (trisomy 13)


 Structural defect of brain
 Scalp defects
 Small eyes (microphthalmia) and other eye defects
 Cleft lip and palate
 Polydactyly
 Cardiac and renal malformations.
MX
 Surgical repair of the lip may be performed within the first week of life for cosmetic reasons,
although some surgeons feel that better results are obtained if surgery is delayed. (3 months)
 The palate is usually repaired at several months of age (6-12 months)

Complication
 Secretory otitis media is relatively common and should be sought on follow-up.
 Infants are also prone to acute otitis media.
 Adenoidectomy is best avoided

Pierre Robin sequence


 Microthalamia
 Microganthia
 Stridor
 Cleft palate
 Cataract

PEDIATRICS MCQ REVIEW – LKSM 12 Page 34


#MMS 33

New born screening

Neonatal screening is performed for, (INTAKE31)


A. Hypothyroidism.
B. Phenylketonuria.
C. Neonatal deafness.
D. Congenital cardiac abnormalities.
E. Developmental dysplasia of hip.
T T T T T

 Congenital Hypothyroidism /G6PD / Congenital adrenal hyperplasia – Guthrie test


 Phenylketonuria/ Cystic fibrosis /Sickle cell anemia – not done in SL
 Neonatal deafness –oto-acustic emission (some centers)
 Congenital cardiac abnormalities – pulse oximeter R/hand and R/leg
 Developmental dysplasia of hip.- newborn physical examination

Routine new born screening in Sri Lanka (INTAKE32)


a. Hearing assessment
b. Hip dislocation assessment
c. Congenital heart disease assessment
d. Hypothyroidism
e. Phenylketonuria

T T T F

Abnormal prenatal screening can be seen in, (INTAKE31)


A. Transposition of great arteries.
B. TOF.
C. ASD.
D. VSD.
E. Co- arctation of aorta.

* Right upper limb and right lower limb to do pulse oximetry screening (mostly after 24
hours)
* In new born baby ductus arteriosus is closed so pre ductal and post ductal should be the
same or slight difference (less than 3%)

If the differential saturation is more than 3% we say differential cyanosis( pre ductal > post ductal)
Lower body gets the mixed blood This is seen in,
 PDA
 PPHN
 LVOT

PEDIATRICS MCQ REVIEW – LKSM 12 Page 35


#MMS 33

Reverse differential cyanosis – post ductal > pre ductal


Causes - Transposition of great arteries

Following must be screened before discharging a neonate after delivery, (INTAKE 29)
A. Cleft palate
B. Deafness
C. Coarctation of the aorta
D. Hip dislocation
E. Cataracts
T F T T T

Defness ? 2 weeks
Red reflex

Neonatal deaths
Which of the following is commonest cause for neonatal deaths in SL(INTAKE32)
a. Extreme prematurity
b. Chromosomal abnormalities
c. Birth asphyxia
d. Neonatal infections

T
Chromosomal abnormalities > LBW > Birth asphyxia > neonatal infections

Neonatal hypoglycemia
3-day old baby boy with 2.2 kg of birth weight presented with poor activity and convulsions. On
examination he was pink, peripheral circulation was good. CBS is 20mg/dl. What is the best
management option?
a) 10% glucose bolus + breast feeding
b) 10% glucose bolus + 10% glucose infusion
c) 5% glucose bolus + infusion
d) 50% glucose bolus + 10% glucose infusion
e) 10% glucose infusion + breast feeding

T
Neonatal hypoglycaemia
•If all readings are >45mg/dl and the measurements can be discontinued
•However, the baby should be observed for a further 24 hours for establishment of feeding
•If any of the readings are 45 mg/dl - 25 mg/dl or less and the baby is asymptomatic, a feed should be
given immediately under supervision and a repeat blood sugar level should be done in 1 hour

PEDIATRICS MCQ REVIEW – LKSM 12 Page 36


#MMS 33

•If the blood sugar level is <25 mg/dl or if the baby is symptomatic within the definition of
hypoglycaemia, intravenous bolus of 10% dextrose 2ml/kg should be given followed by an infusion of
10% dextrose 60ml/kg/day .The concentration of infused dextrose can be increased up to 12.5% via a
peripheral vein or up to 15% via a central vein

6 day old baby presented with poor feeding, lethargy and floppy. He had cold extremities andCRFT > 2
sec. He was given 2 bolus of N/S .
 CRP 1mmol/L
 Na 125mmol/L
 K 6.9 mmol/L
 CBS 3mol

What is the next best step in the management? (INTAKE32)


a. 10% dextrose bolus
b. 3% NS bolus
c. IV antibiotics
d. IV hydrocortisone
e. IV fludrocortisone

A 4 week old baby presented with poor feeding and poor weight gain. Birth weight was 3.1 kg and
current weight is 2.9 kg. Examination is normal. Investigations results are given below. What is the next
best investigation? (COL2016)
WBC - 8.8*103 N - 48% L - 47%
Na - 121 mmol/L
K - 5.3 mmol/L
Urea - 4 mmol/L ( 7 to 20 mg/dL)
a) ABG
b) Estimation of anion gap
c) Random blood sugar

T
Poor feeding and poor weight gain can be due to sepsis

Term new born and preterm


A term new born baby,
A. Moves the head in prone position.
B. Able to flex the hip and knee.
C. Moro reflex is positive.
D. Follows a moving object in 180 degree by eyes.
E. If develop hiccups should be investigated immediately.
F T T F

PEDIATRICS MCQ REVIEW – LKSM 12 Page 37


#MMS 33

Reflexes
Primitive reflexes
 Moro
 Grasp
 Rooting
 Stepping response
 Asymmetrical tonic neck reflex

Postural reflexes
 Labyrinthine righting
 Postural support
 Lateral propping
 Parachute

In comparison with a term baby, a preterm baby (COL2016)


a) Has a higher risk of developing intracranial haemorrhages
b) Is at a higher risk of getting iron deficiency
c) Is more prone to get hypothermia
d) Needs more ml per kilogram of fluid
e) Is at a higher risk of developing osteopenia

T T T T T o electrolyte imbalance
o osteopenia of prematurity
Medical problems of preterm  Nutrition
 Need for resuscitation and stabilization
 Infection
at birth
 Jaundice
 Respiratory:
 Intraventricular
o respiratory distress syndrome
haemorrhage/periventricular
o pneumothorax
leukomalacia
o apnoea and bradycardia
 Necrotizing enterocolitis
 Hypotension
 Retinopathy of prematurity
 Patent ductus arteriosus
 Anaemia of prematurity
 Temperature control
 Iatrogenic
 Metabolic:
 Bronchopulmonary dysplasia (BPD)
o hypoglycaemia
 Inguinal hernias.
o hypocalcaemia

WOTF needs further assessment in a 2-day old baby, (COL2016)


a) Pink colour patch in nappy
b) Red colour mark in the back of neck
c) Bluish macule in the buttocks
d) Passing of black colour stools
e) Greenish vomitus

PEDIATRICS MCQ REVIEW – LKSM 12 Page 38


#MMS 33

T  Palpable and large bladder


 Palpable and large bladder
 Greenish vomiting – bile can be due to
intestinal obstruction so urgent
investigation should be done
NON significant abnormalities
 Peripheral cyanosis of the hands and
feet
 Traumatic cyanosis
 Swollen eyelids and distortion of shape
of the head
 Subconjunctival haemorrhages
 Small white pearls (Epstein pearls
 Cysts of the gums (epulis) or floor of the
mouth (ranula).
 White vaginal discharge or small
withdrawal bleed in girls.
 Capillary haemangioma or ‘stork bites’
 Neonatal urticaria (erythema toxicum) –
white pinpoint papules at the centre of
an erythematous base
 Milia
 Mongolian blue spots
 Umbilical hernia - usually resolves
within the first 2–3-years.
 Positional talipes
 Caput succedaneum
 Cephalhaematoma
 Urate crystals diaper - dehydrated
 Malena – black stools

Significant abnormalities
 Port-wine stain (naevus flammeus).
 Strawberry naevus (cavernous
haemangioma).
 Natal teeth consisting of the front lower
incisors
 Extra digits
 Heart murmur
 Midline abnormality over the spine or
skull,

PEDIATRICS MCQ REVIEW – LKSM 12 Page 39


#MMS 33

Neonatal jaundice

New born baby presented with jaundice at 36 hours of life. Blood picture shows normocytic red cells,
Direct Coomb’s Test : Positive, Reticulocyte count is 20%. Blood group of mother is O Positive and
of baby is O Negative. What is the most possible cause for jaundice? (INTAKE 29)
A. ABO incompatibility
B. Rh incompatibility
C. Congenital spherocytosis
D. G6PD deficiency
E. Minor blood group incompatibility
T

G6PD – blister cells

A term baby of O+ was born with a birth weight of 2.5 kg to a mother of A+ and was found to be
pale. His haemoglobin was 10 g/dL, reticulocyte count 5% and PCV 34. Which of the following is the
most likely diagnosis? (COL2016)
a) ABO incompatibility
b) Foetomaternal haemorrhage
c) Iron deficiency anaemia
d) Physiological anaemia
e) G6PD deficiency

T
ABO incompatibility - Haemolysis can cause severe jaundice but it is usually less severe than in
rhesus disease. The infant’s haemoglobin level is usually normal or only slightly reduced and, in
contrast to rhesus disease, hepatosplenomegaly is absent. The direct antibody test (Coombs’ test),
which demonstrates antibody on the surface of red cells, is positive. The jaundice usually peaks in
the first 12 hours to 72 hours

Extra hepatic biliary atresia(INTAKE 30)


1. increase urine urobilinogen
2. diagnosis only with USS
3. present 1st week of life
4. Causes bilirubin encephalopathy
5. Presents first week of life
T

Biliary atresia is a cause for conjugated hyper bilirubinemia


And Cause for prolong jaundice

USS – contracted or absence GB


HIDA - excretion to intestine is absent
Confirmed by cholangiogram

Bilirubin encephalopathy is caused by unconjugated bilirubin

30 hrs old baby receiving double phototherapy for jaundice. Feeding and active well. Mother had
ruptured membranes for 18 hrs prior delivery. Mother O negative baby B positive. Coombs weekly

PEDIATRICS MCQ REVIEW – LKSM 12 Page 40


#MMS 33

positive. Blood picture hemolytic. Most suitable diagnosis(INTAKE 30)


a. early onset sepsis
b. Rh hemolytic disease
c. abo hemolytic disease
d. hereditary spherocytosis
e. G6PD deficiency
T

ABO incompatibility – weekly coombs positive


HS and G6PD Coombs negative

ABO incompatibility NOT happen in AB mother or O baby


EOS – risks
 PROM >17 hrs
 Prolong labor > 24 hrs
 Low birth weight
 Chorioamnitis
 Maternal pyrexia

4 days old baby presented with icterus and pale, was suspected to have G6PD deficiency.Laboratory
findings could be, (INTAKE 29)
A. Increased direct bilirubin
B. Sperocytes in blood picture
C. Increased reticulocyte count
D. Hemoglobin in urea
T

Increase indirect bilirubin


Cause for jaundice within 24-48 hrs of life
Others are ABO /RH incompatibility and hereditary spherocytosis

Respiratory distress
A new born baby on treatment with surfactant for surfactant deficient lung disease suddenly
deteriorated with a drop of saturation to 75%. On examination, chest expansion and air entry is
reduced on the right side. Endotracheal tube is in the correct position. What's the most appropriate
next step in the management? (COL2021)
a. Administer another surfactant dose
b. Increase oxygen delivery (FiO₂)
c. Increase peak inspiratory pressure (PIP)
d. Perform a needle thoracostomy
e. Suck out the secretions through the endotracheal tube

T
Complications of procedure
 plugging of endotracheal tube (ETT) by surfactant
 haemoglobin desaturation and increased need for supplemental O2
 bradycardia due to hypoxia
 tachycardia due to agitation, with reflux of surfactant into the ETT

PEDIATRICS MCQ REVIEW – LKSM 12 Page 41


#MMS 33

Regarding surfactant deficient lung disease, (COL2021)

A. Ground glass appearance in the chest X-ray is seen


B. Respiratory distress occurs within 4 hours of life
C. Antenatal steroids reduce the risk
D. Congenital pneumonia is a differential diagnosis
E. The risk of surfactant deficiency increases with an IUGR baby

T T T T F

Respiratory distress syndrome


 Common before 28 weeks (preterm)
 More in boys
 Rare in term but in infants with DM mother
 Surfactant makes from 20th week /optimize at 35th week
 X-ray - Homogeneous opaque infiltrates and air Broncho grams
 Tx – NBM /surfactant therapy (reduce mortality and morbidity )
 Risk
o Decrease the risk of RDS -Antenatal steroid use / PROM
o Increase risk of RDS – factors that cause increase risk of pre mature birth*
*Any case that tense the uterus - Macrosomia /multiple pregnancies
Congenital pneumonia
 risk are PROM ,chorioaminitis ,foetal hypoxia
 Caused by group B streptococcus (it is EOS )
 X-ray – patchy shadowing and consolidation
 TX – benzyl penicillin

PEDIATRICS MCQ REVIEW – LKSM 12 Page 42


#MMS 33

Prenatal infections
Regarding intrauterine infections, (COL2016)
a) HIV transmission directly correlated to maternal viral load
b) Zika virus causes microcephaly

T F
Zika virus
 transmitted by Aedes mosquitoes,
 Central and South America and the Caribbean
 Children - fever, rash, conjunctivitis, and arthralgia
 pregnancy - associated with fetal microcephaly and other brain abnormalities

HIV
 Vertical transmission of HIV can occur before (intrauterine), during (intrapartum), or after
delivery (through breast-feeding ).
 Several studies have shown an increased rate of transmission in women with advanced
disease (i.e., AIDS) or high viral load ( >50,000 copies/mL), some transmitting mothers in
each group were asymptomatic or had a low, but detectable, viral load. Thus, in the USA it is
recommended to consider cesarean section if the viral load is >1,000 copies/mL.

Maternal health
Maternal charasteristic which predicts birth weight of baby best(INTAKE32)
a. pre pregnancy weight
b. height
c. BMI
d. maternal weight gain during pregnancy
e. iron

PEDIATRICS MCQ REVIEW – LKSM 12 Page 43


#MMS 33

GROWTH AND PUBERTY

Growth
A 1-year-old boy was referred to you with concerns about growth. At birth, his weight and height
were plotted on the 75th centile. His weight and length fall gradually reaching the 50th centile over
the first 6 months of life. Thereafter his growth continued along the 50th centile. The child’s
development is normal and he is on a balanced diet. Which of the following is the best explanation
for this scenario? (COL2021)
a. Acquired hypothyroidism
b. Catch down growth
c. Coeliac disease
d. Constitutional growth faltering
e. Failure to thrive

‘Catch-down’ growth
 An additional diagnostic problem is ‘catch-down’ (as opposed to ‘catch-up’) weight.
 This is when an infant’s weight falls from the birth centile, which is determined by the
intrauterine environment, to a lower, genetically determined growth centile.
 These infants need only close monitoring of their growth over a few months.
 While children with recent-onset failure to thrive usually maintain their height, this may
become compromised by prolonged, inadequate weight gain.
 The child’s developmental progress may also be adversely affected.

× Mild failure to thrive being a fall across two centile lines and severe being a fall across
three centile lines
× A weight below the 0.4th centile should always trigger an evaluation

Short stature
Short stature is seen in, (COL2021)
B. Turner syndrome
C. Congenital hypothyroidism
D. Down's syndrome
E. Klinefelter syndrome
F. Marfan syndrome

T T F F

SS definition – height below <2SD


 Turner’s syndrome Short stature is a cardinal feature

PEDIATRICS MCQ REVIEW – LKSM 12 Page 44


#MMS 33

Hypothyroidism causing SS
 Usually caused by autoimmune thyroiditis during childhood
 This produces growth failure, usually with excess weight gain.
 Congenital hypothyroidism is diagnosed soon after birth by screening and so does not result
in any abnormality of grow

14 yrs old girl presented to the pead clinic due to short stature. She was the shortest in her class
since pre school.Her development is age appropriate. School performance is below average On
examination she was pre pubertal and had a heart murmur. What is the most appropriate e
investigation to identify the diagnosis? (INTAKE32)
a. TSH
b. FSH
c. ECHO
d. Karyotyping
e. MRI brain

Commonest cause for moderate short stature (-2SD) globally; (INTAKE32)


a. Psycho- social deprivation
b. Familial short stature
c. Protein energy malnutrition
d. Low birth weight
e. Recurrent infections

Short stature in a 14 years old child, (INTAKE31)


A. Klinefelter xd
B. Crohn’s disease
C. Psychosocial deprivation.
D. Chronic malnutrition.
E. Constitutional delay.

T T T T

Klinefelter xd (47 XXY and XXY karyotype)


 Infertility – most common presentation
 Hypogonadism with small testes
 Pubertal development may appear normal(some males benefit from testosterone therapy)
 Gynaecomastia in adolescence
 Tall stature
 Intelligence usually in the normal range, but some have educational and psychological
problems

PEDIATRICS MCQ REVIEW – LKSM 12 Page 45


#MMS 33

Cause for SS

 Familial – Following growth centile within predicted range for parental height
 IUGR - Short from birth (but normal mid parental height)
 Endocrine – hypothyroidism / Pan hypopituitarism (secondary GH deficiency) / rickets -
Falling off height centiles. Weight centile >height centile. i.e. short and overweight Markedly
delayed bone age.
 Syndrome – Noonan /downs /Turner /Russel Silver Syndrome /Laron syndrome
 Psychosocial deprivation and Chronic malnutrition - Falling off height centiles. Weight centile
< height centile. Delayed bone age
 Constitutional delay- Short stature accentuated by delayed puberty. Delayed bone age
 Extreme short stature- rare
 Disproportion - skeletal dysplasia – legs>back / storage disorders – back>legs

Chronic illness causing SS


o coeliac disease
o chron’s disease
o CKD
o Cystic fibrosis
o Congenital heart disease

Precocious puberty
In precocious puberty(INTAKE32)
a. Bone age is delayed
b. Causes reduction in final height
c. Congenital adrenal hyperplasia is a cause
d. In pituitary tumors, normal pubertal development occurs
e. Is the development of secondary sexual characteristics before 8 years in girls

F F T F T
Onset of puberty before 8 years in girls and before 9 years on males

PEDIATRICS MCQ REVIEW – LKSM 12 Page 46


#MMS 33

PEDIATRICS MCQ REVIEW – LKSM 12 Page 47


#MMS 33

NUTRITION

Breast feeding
A 10 days old baby girl presented with reduced urine output. She was born at term with a birth
weight of 2.9kg and the current weight is 2.6kg. She suckles from her mother’s breast for 30
minutes. What's the most appropriate next step in the management of this baby? (COL2021)
a) Collect urine for culture
b) Observe and correct breastfeeding technique
c) Prescribe a lactogogue to the mother
d) Top up with expressed breast milk
e) Top up with formula feeds

 ? Adequate milk – urine output is normal


 Baby is usually Usually 10-20 min breast feed . if he take more than that it can be due to
problem of technique

Compared to cow’s milk, breast milk has (COL2016)


a) more iron
b) more protein
c) high whey : casein ratio
d) low sodium
e) low PUFA

F F F T T
PUFA - Polyunsaturated fatty acids

Breast milk differ from cow milk by, (INTAKE 29)


A. High lactose
B. High fat
C. Low Sodium
D. Low Potassium
E. Low proteins
T F T T T

Breast milk Cow milk


Lactose ↑
Fat ↑
Na ↑
K ↑
Proteins ↑
Energy ↑
iron ↑

PEDIATRICS MCQ REVIEW – LKSM 12 Page 48


#MMS 33

Complementary feeding
Most important micronutrient that should be included in formula milk meant for 6M to 12M(INTAKE32)
a. vitamin A
b. zinc
c. calcium
d. iron
e. iodine

T
Following food are not best recommended for complementary foods, (INTAKE 29)
A. Food with bran
B. Low fat yogurt
C. Bee honey
D. Soft boiled egg
E. Raw milk
T

It is done gradually, initially with small quantities of pureed fruit, root vegetables, or rice. If weaning
takes place before 6 months of age, wheat, eggs and fish should be avoided. Foods high in salt and
sugar should also be avoided and honey should not be given until 1 year of age because of risk of
infantile botulism.

Most compelling reason to start formula feeding in children is(INTAKE 30)


1. Mother having tuberculosis
2. Mother having heart failure
3. Mother having HIV
4. Baby having cleft lip and cleft palate
5. Baby having phenylketonuria
T

Indication for formula feeding


 Maternal death /surrogate mother
 Maternal illness HIV* /cardiac failure /psychiatric illness
 Working mother
 Mother unwilling to breast fed
 Lactation failure
 Ill mother
 Mother unavailable
 Brest milk inadequate
 Lactulose intolerance

* HIV mothers with low viral loard can continue BF

PEDIATRICS MCQ REVIEW – LKSM 12 Page 49


#MMS 33

Iron deficiency
A 14-month-old girl presented with difficulty in gaining weight for 6 months. She is on a
complimentary diet containing rice, yams, and vegetables. She has sleep disturbances and wakes
up at night for frequent breastfeeding. On examination, she is mildly pale but well active. Her
weight to height is between -2SD and -3SD. What is the most appropriate next step? (COL2021)
a) Add formula feed to current breastfeed
b) Give micronutrient supplements
c) Improve the quality of complementary food
d) Prescribe an appetite stimulant
e) Refer for nutritional rehabilitation

The main causes of iron deficiency


 Inadequate intake
 Malabsorption
 Blood loss.

Iron may come from:


 Breastmilk (low iron content but 50% of the iron is absorbed)
 Infant formula (supplemented with adequate amounts of iron)
 Cow’s milk (higher iron content than breastmilk but only 10% is absorbed)

Iron deficiency may develop because of a delay in the introduction of mixed feeding beyond 6
months of age or to a diet with insufficient iron-rich foods, especially if it contains a large amount
of cow’s milk
Iron absorption is markedly increased when eaten with food rich in vitamin C (fresh fruit and
vegetables) and is inhibited by tannin in tea

Most important cause of pica in childhood(INTAKE 30)


1. Hook worm
2. Iron deficiency anemia
3. Lead poisoning
4. Zn deficiency
5. Mental retardation
T

Zn deficiency
Zn deficiency, (COL2021)
A. Causes immunodeficiency
B. Causes chronic diarrhoea
C. Causes acral dermatitis
D. Is a cause for macrocytic anaemia
E. Is treated with micronutrient sachets

PEDIATRICS MCQ REVIEW – LKSM 12 Page 50


#MMS 33

F T T F F
Acrodermatitis enteropathica is an inherited disorder caused by malabsorption of zinc.
Infants develop growth retardation, severe diarrhoea, hair loss and a skin rash, which can occur
anywhere on the body, but most often around the mouth, genitalia and
Oral ZN is the treatment for Zn deficiency
Micro nutrient sachet
o Composition
 Vitamin A, C, D, E B1, B2, B3, B6, B12 (NO vitamin K)
 Folic acid
 Iron
 Zink
 Copper
 Selenium
 Iodine
 It gives all children to prevent iron deficiency anaemia after 1st 6 month of life
 Given at age of 6, 12 and 18 months
 At one round give 30 packets
 After that give another 30. Altogether 60 days should be complete continuously

Macrocytic anaemia – folate / vitamin B12

Vitamin deficiencies
Correctly matched, (INTAKE31)
A. Acanthosis nigricans –simple obesity.
B. Zn deficiency – acrodermapathy.
C. Kowshiakor – flacky paints
D. Fatty acid deficiency -??
E. Thiamine deficiency – pellagra.
T T F

B1(thiamine) – beriberi /wernike-kroscroff


B2 – angular stomatitis
B3 (niacin)–pelagra
C –scurvy
E-ataxia
Zn deficiency – acrodermatitis enteropathica

Acrodermapathy is a feature of thyroid disease

First ever nutritional deficiency described by clinically is(INTAKE 30)


1. kwashiorkor
2. scurvy Vit C
3. berry berry B1
4. pellagra B3
5. exophthalmia Vit A

PEDIATRICS MCQ REVIEW – LKSM 12 Page 51


#MMS 33

Fruit which is richest in vitamin C and found in Sri Lanka, (INTAKE 29)
A. Amberalla
B. Nelli
C. Guava
D. Lime
E. Veralu
T

Anthropometry measurements
A 6-year old girl has contractures of the arms and legs, cannot stand, but can sit upright. Which of
the following measurements would be estimaiimators of height? (INTAKE32)
a. Sitting height
b. Arm span
c. Mid arm muscle circumference
d. Crown rump length

T F F

B/C contractures cant

In a temporary settlement camp for displaced persons the MOH of the area is expected to assess the
<5 year population, for the assessment of nutritional status. What is the best anthropometric
measure for this screening? (COL2016)
a) Chest circumference
b) Height or length
c) Mid arm circumference
d) Skin fold thickness
e) weight

Growth and nutrition are closely related, such that the mean height of a population reflects its
nutritional status.
Mid arm circumference - Which is related to skeletal muscle mass, can be measured easily and
repeatedly and is independent of age in children aged 6 months to 5 years. It is especially useful for
screening children for malnutrition in the community

PEM
15 month old child diagnosed with protein energy malnutrition. Best management step is, (INTAKE 29)
A. Fluid resuscitation
B. Start on NG feeding
C. Look for socioeconomic aspects

PEDIATRICS MCQ REVIEW – LKSM 12 Page 52


#MMS 33

D. Psychological stimulation
E. Start on micronutrients
T

Regarding chronic malnutrition, (INTAKE 29)


A. Height is more affected than weight
B. Height to weight ratio is better in chronic malnutrition
C. Direction of the curve in growth chart is more important than the position
T F T

weight for age <2SD – under weight


height for age Chronic <2SD – stunted
malnutrition
weight for height Acute malnutrition <2SD – wasted
<3SD – severely wasted (Marasmus)
presence of symmetrical oedema (Kwashoiorkor)
combination of mild oedema and wasting (Marasmic
Kwashiorkor)
MUAC Acute malnutrition cut-off <11.5cm

Obesity
Leading cause for global child obesity, (INTAKE 29)
A. Carbonated drinks
B. Fast food
C. Ice cream
D. Chocolate
E. Milk industry

T
High fat fast food with processed food

Commonest nutritional disorder when Asians migrate to United Kingdom(INTAKE31)


A. Xerophthalmia.
B. Rickets
C. Scurvy
D. Obesity
E. Marusmus
T

Regarding simple obesity in childhood (INTAKE32)


a. Bone age is delayed
b. Leads to DM 1
c. Leads to obesity in adult life
d. Best measured using BMI
e. Measured using BMI in usual clinical practice

PEDIATRICS MCQ REVIEW – LKSM 12 Page 53


#MMS 33

F F T T T
In children, the Body Mass Index (BMI) is (weight in kg/(height in m)2 ) and is expressed as a BMI
centile in relation to age-matched and sex-matched population

12 year old boy presented with day time somnolence. His school performance is good. Weight is 56
kg. Most likely diagnosis is, (COL2016)
a) Hypothyroidism
b) Obstructive sleep apnoea
c) Substance abuse
d) Studying late into the night
e) Lethargy

Weight calculations
 Infants < 12 months: Weight (kg) = (age in months + 9)/2
 Children aged 1-5 years: Weight (kg) = 2 x (age in years + 5)
 Children aged 5-14 years: Weight (kg) = 4 x age in years.

Complications of obesity
• Orthopaedic – slipped upper femoral epiphysis, tibia vara (bow legs), abnormal foot
structure and function
• Idiopathic intracranial hypertension (headaches, blurred optic disc margins)
• Hypoventilation syndrome (daytime somnolence, sleep apnoea, snoring, hypercapnia,
heart failure)
• Non-alcoholic fatty liver disease
• Gall bladder disease/gallstones
• Polycystic ovarian syndrome
• Type 2 diabetes mellitus
• Hypertension
• Abnormal blood lipids
• Other medical sequelae, e.g. asthma, changes in left ventricular mass, increased risk of
certain malignancies (endometrial, breast, and colon cancer)
• Psychological sequelae – low self-esteem, teasing, depression

Mother of 6 year old boy complains of day time sleepiness and loud snoring at night. He has dark
pigmentation in his neck and axilla. His performance at school is good. Weight of the child is 54kg.
Whatis the most probable diagnosis? (INTAKE31)
A. Hypothyroidism
B. Obstructive sleep apnoea
C. Substance abuse
D. Studying at night
E. Cushing’s disease
T

PEDIATRICS MCQ REVIEW – LKSM 12 Page 54


#MMS 33

GIT AND LIVER

IBD
A 15-year-old boy is being evaluated for delayed puberty and short stature. Mid parental height is
on the 75th centile. He has recently developed a painful erythematous nodule over the right shin
which has resolved now. On examination, he is thin and pale and has aphthous ulcers. What is the
most likely diagnosis? (COL2021)
a. Coeliac disease
b. Giardiasis
c. Crohn's disease
d. Irritable bowel syndrome
e. Ulcerative colitis

T
Extra intestinal manifestation of IBD
 Skin
o Clubbing
o Erythema nodosum
o Pyoderma gang (esp. UC)
 Eyes
o Iritis
o Episcleritis
o Conjunctivitis
 Joints
o Arthritis (non-deforming, asymm)
o Sacroiliitis
o Ank spond
 HPB
o PSC + cholangiocarcinoma (esp. UC)
o Gallstones (esp. Crohn’s)
o Fatty liver
 Other
o Amyloidosis
o Oxalate renal stones (esp. Crohns)

PEDIATRICS MCQ REVIEW – LKSM 12 Page 55


#MMS 33

CLCD
Clinical features of chronic liver disease in children include, (COL2021)
a) Intentional tremor
b) Finger clubbing
c) Telangiectasia
d) Palmar erythema
e) Grey nails

F T T T

 Hands
o Clubbing (± periostitis)
o Leuconychia (↓ albumin)
o Terry’s nails (white proximally, red distally)
o Palmer erythema
o Dupuytron’s contracture
 Face
o Pallor: anemia of chronic disease
o Xanthelasma: primary biliary cholangitis
o Parotid enlargement (esp. with EtOH)
 Trunk
o Spider naevi (swallowan blood vessels is a type Telangiectasia) (>5, fill from centre)
o Gynaecomastia
o Loss of 2O sexual hair
 Abdomen
o Striae
o Hepatomegaly (may be small in late disease)
o Splenomegaly
o Dilated superficial veins (Caput medusa)
o Testicular atrophy

Gastroenteritis
Absolute indication/s for IV fluid therapy in gastroenteritis(INTAKE32)
a. Vomiting
b. Paralytic ileus
c. Diarrheal loss of >10mg/kg
d. Secondary lactose intolerance
e. shock

T F F F T

PEDIATRICS MCQ REVIEW – LKSM 12 Page 56


#MMS 33

2-year-old boy presented with high fever and bloody diarrhoea with abdominal pain on
examination there was generalized abdominal tenderness. Which of the following are possible
causative organisms? (COL2016)
a) Campylobacter
b) Clostridium botulinum
c) Enterotoxigenic e. coli
d) Giardia lamblia
e) Shigella dysenteriae

T F F F T

Causes for bacillary dysentery


 Escherichia coli (entero invasive ,hemorrhagic)
 Salmonella
 Shigella shigai ,shigella flexineris
 Campylobacter jejuni – NO common in SL

Giardia and Cryptosporidium gastroenteritis


 Sudden change to loose or watery stools often accompanied by vomiting
 Contact with a person with diarrhoea and/or vomiting or recent travel abroad. A

2 year old boy presented with high fever and bloody diarrhea with abdominal pain on examination
there was generalized abdominal tenderness. Which of the following are possiblecausative
organisms?
1. Shigella dysenterie
2. Giardia lambia
3. Clostridium botulinum
4. Entero Toxic E. coli
5. Campylobacter
T F F F T

Salmonella, E coli entero invasive and hemorrhagic

A 1-year-old boy presented to the clinic with intermittent stools containing undigested food
particles for 3months duration. No fever, no loss of appetite nor loss of weight, no vomiting. He is
thriving well. Abdominal examination is normal. Most appropriate management of this child? (COL2021)
a. Stool full report
b. Prescribe probiotics
c. Reassure and discharge
d. Lower GI endoscopy
e. Measure serum calprotectin levels

T
Chronic diarrhoea
 In an infant with faltering growth, consider coeliac disease and cow’s milk protein allergy.
 Following bowel resection, cholestatic liver disease or exocrine pancreatic dysfunction,
consider malabsorption.

PEDIATRICS MCQ REVIEW – LKSM 12 Page 57


#MMS 33

 In an otherwise well toddler with undigested vegetables in the stool, consider chronic non-
specific diarrhoea.( Toddler diarrhoea)

Constipation
Causes for Constipation(INTAKE32)
a. Fissures
b. Child abuse
c. Hirchsprung disease
d. Hypothyroidism
e. Spina bifida

T T T T T

Hurshprung disease(INTAKE 30)


1. Present with delayed passage of meconium.
2. Due to albescence of parasympathetic ganglions in gut.
3. Common incidence in downs.
4. gush of feces will comes out following DRE
5. Commonly complicated with enterocolitis.
T T T T F

Usually present with delayed passage of meconium – DDs are imperforated anus / Hirschsprung’s
disease

Hirschsprung’s disease
* Absence of awarback ganglion
o Enteric – awerbark (mynrtic ) and meisner
o Awerbark – sympathetic and parasympathetic
o Submucosal misner – parasymatathatic only
* Common in downs – on examination fecal masses should palpate
* 75% common in recto sigmoid SO after DRE obstruction will release .pass stools and flatus
* Complications – occasionally present with enterocolitis / growth failure /chronic
constipation

Lactose intolerance
Secondary lactose intolerance seen commonly in(INTAKE32)
a. Cows milk allergy
b. Hook worm infestation
c. Rotavirus gastroenteritis
d. Shigella dysentery
e. Bacillary dysentery

PEDIATRICS MCQ REVIEW – LKSM 12 Page 58


#MMS 33

Intussusception
7 year old boy presented with intermittent abdominal pain. From this morning started having severe
abdominal pain with drawing leg on to chest. He has become pale with dissented abdomen. He had
breakfast and now reluctant to touch abdomen. What is the most important investigation, (INTAKE 29)
A. Plain X-ray abdomen
B. Barium enema
C. Barium meal
D. USS
E. 99Tc scan
T

Usually occurs between 3 months and 2 years of age


Clinical features
* Paroxysmal, severe colicky pain and pallor – during episodes of pain, the child becomes pale,
especially around the mouth, and draws up his legs. He initially recovers between painful
episodes, but subsequently becomes increasingly lethargic
* May refuse feeds, may vomit, which may become bile-stained depending on the site of the
intussusception
* A sausage-shaped mass – often palpable in the abdomen (Fig. 13.6b)
* Passage of a characteristic redcurrant jelly stool comprising blood-stained mucus – this is a
characteristic sign but tends to occur later in the illness and may be first seen after a rectal
examination
* Abdominal distension and shock
Investigation
* An X-ray of the abdomen may show distended small bowel and absence of gas in the distal
colon or rectum. Sometimes the outline of the intussusception itself can be visualized.
Abdominal ultrasound is helpful both to confirm the diagnosis and to check response to
treatment
Management
* Reduction is attempted by rectal air insufflation unless peritonitis is present
* Surgery is required if reduction with air is unsuccessful or for peritonitis

*****99TC – used to diagnose Michel’s diverticulum gastric mucosa *****

3-month-old baby on exclusive breastfeeding presented with vomiting and diarrhoea for 3 days. He
is not dehydrated. Well looking. Perianal excoriations present. Stools were positive for reducing
substances. What advice would you give you to the mother? (COL2016)
a) Continue breastfeeding
b) Prescribe lactose free formula
c) Check stools for reducing substances
d) Evaluate for cow milk protein allergy
e) Reassure parents

PEDIATRICS MCQ REVIEW – LKSM 12 Page 59


#MMS 33

Intestinal obstruction
A 5 day old baby presented with bilious vomiting on day 5. Baby had passed meconium on birth.
There was no abdominal distension. CRFT <2 sec. Baby was pink and well. What is the diagnosis?
(COL2016)
a) Duodenal atresia
b) Necrotizing enterocolitis
c) Jejunal atresia
d) Septicemia
e) Volvulus

jejunoileal atresias
 Polyhydramnios is identified in 20-35% of jejunoileal atresias, and it may be the first sign of
intestinal obstruction
 Abdominal distention is rarely present at birth, but it develops rapidly after initiation of
feeds in the first 12-24 hr.
 Distention is often accompanied by vomiting, which is often bilious. Up to 80% of infants
fail to pass meconium in the first 24 hours of life.
 Jaundice, associated with unconjugated hyperbilirubinemia, is reported in 20-30% of
patients.

Malrotation
• Uncommon but important to diagnose.
• Usually presents in the first 1–3 days of life with intestinal obstruction from Ladd bands
obstructing the duodenum or volvulus.
• May present at any age with volvulus causing obstruction and ischaemic bowel.
• Clinical features are bilious vomiting, abdominal pain and tenderness from peritonitis or
ischaemic bowel.
• An urgent upper gastrointestinal contrast study is indicated if there is bilious vomiting.
• Treatment is urgent surgical correction.

PEDIATRICS MCQ REVIEW – LKSM 12 Page 60


#MMS 33

INFECTION/ IMMUNITY

Dengue
5 year old boy presented with fever for 3 days. Fever 39.3, PR 140, BP 85/60, right hypochondriac
tenderness and reduced air entry to right lung base. First step of themanagement is, (INTAKE 30)
a. FBC
b. PCV
c. USS Abdomen
d. 10 mg/kg N saline
e. 10 mg/kg dextran
T

* Child is in Shock/ critical phase of dengue


* So ABC approach
* In C should take blood for investigation (ABCS)
* In ward easily available one is PCV
* DO PCV as a base line and continue treatment according to the phase

10 year old baby admitted with fever, body aches and headache for 4 days. On examinationafebrile,
conscious and alert. BP 90/77mmHg,, PR 140 BPM,, PCV 52 and reduced air entry inright side. Next
step in management? (INTAKE31)
A. Blood transfusion
B. 10ml/kg 0.9 saline bolus
C. 10ml/kg 0.9 saline + 5% dextrose bolus
D. 10ml/kg dextrose bolus
E. 20ml/kg 0.9 saline bolus
T

12year old girl diagnosed and admitted with DHF is currently on the 20th hour of critical
phasemanagement. She complains of abdominal pain and is restless. A 10ml/kg normal saline bolus
wasgiven the PCV before and after the saline bolus was 52 and 38 respectively.The vitals after the
bolus were:
PR 142/min,
BP 80/70 mmhg,
CRFT 6 seconds.
UOP was 0.28ml/kg/hr.
what’s the most appropriate step in the management of this patient? (INTAKE32)
a. Blood transfusion 5ml/kg
b. Dextrose saline 7ml/kg/hr
c. Dextran bolus 10ml/kg/hr
d. Calcium gluconate infusion
e. Repeat normal saline bolus over 20 minutes

PEDIATRICS MCQ REVIEW – LKSM 12 Page 61


#MMS 33

Typhoid
Regarding typhoid fever(INTAKE 30)
1. Organism is isolated in blood during first week
2. SAT in positive during first week
3. Chloramphenicol is the first line treatment
4. Chronic carrier state is treated with amoxicillin
5. Notification is done to the MOH of the residential area
T F F F T

Isolate the organism is the gold stranded


 Blood culture within 2 weeks
 Stool culture after 2 weeks

SAT /Widal test – detection of O antigen and H antigen (non specific test ) 1st week diagnosis will be
difficult

Treatment – IV /oral cotrimaxasole, ampicillin, ciprofloxacin (older children)

PEDIATRICS MCQ REVIEW – LKSM 12 Page 62


#MMS 33

Chronic career – treated with ciprofloxacin / if can give according to antibiotic sensitivity

Notifiable diseases
GP A – yellow fever /plague /cholera
GP B – typhoid /TB /gastroenteritis / meningitis /dengue

A 7-year-old boy presents with a high fever and headache for 7 days. No travel history outside of
Colombo. On examination, he was febrile, there was hepatosplenomegaly and a maculopapular
rash. No lymphadenopathy.PR 90/min BP 90/70 mmHg FBC Hb normal / WBC 4 Neutrophils 30%
lymphocytes 70% / Platelets – normal. Most likely diagnosis? (COL2021)
a) Infectious mononucleosis
b) Leptospirosis
c) Systemic onset JIA
d) Typhoid fever
e) Typhus

Maculopapular rash
 Viral
o HHV6
o Parvovirus
o Measles
o Rubella
 Bacterial
o Scarlet fever (group A streptococcus )
o Erythema marginatum – rheumatic fever
o Typhoid fever
o Lyme disease
 Other
o Kawasaki
o JIA

Typhoid fever
 A child with worsening fever, headaches, cough, abdominal pain, anorexia, malaise and
myalgia may be suffering from infection with Salmonella typhi or paratyphi.
 Gastrointestinal symptoms (diarrhoea or constipation) may not appear until the second
week.
 Splenomegaly, bradycardia and rose-coloured spots on the trunk may be present.
 The serious complications of this disease include gastrointestinal perforation, myocarditis,
hepatitis and nephritis.
 IX - Leukopenia /Blood culture Urine or stool culture
 The recent increase in multi-drug resistant strains, particularly from the Indian subcontinent,
means that treatment with cotrimoxazole, chloramphenicol or ampicillin may be
inadequate. A third-generation cephalosporin or azithromycin is usually effective

PEDIATRICS MCQ REVIEW – LKSM 12 Page 63


#MMS 33

Systematic JIA
 May present with continued high grade fever
 Rash transient with fever
 Initially may have arthralgia/myalgia and NO arthritis - later develops - polyarthritis
 generalized lymphadenopathy
 hepatosplenomegaly
 Serositis( pleurisy /pericarditis)
 Anaemia
 Thrombocytosis
 High CRP/ESR

Rabies
Hyper immune anti rabies serum is indicated in(INTAKE32)
a. Face bite by a stray dog
b. Superficial bite by a domestic dog who has fully completed vaccine
c. Multiple bite by a domestic dog who was vaccinated once recently.
d. Hand bite by stray dog
e. Deep leg bite by a mongoose

T F T T T

13 year old child presented with dog bites (multiple) including face. Immediate management include,
(INTAKE 29)

A. Wash with soap and water


B. Infiltrate the wounds with Rabies immunoglobulin
C. Apply tight bandage/dressing
D. Start IV IG
T T F F

Varicella
12yr old boy who is a diagnosed pt with ALL is now on chemotherapy in the ward. His bystandergot
chickenpox..What is the best management option to prevent the child from getting the infection?
(INTAKE32)
a. stop chemotherapy and keep under observation
b. administer iv Aciclovir
c. administer iv immunoglobulin
d. administer Varicella vaccine
e. administer vaccine and immunoglobulin

T
High risk immune suppressed – give IVIG

PEDIATRICS MCQ REVIEW – LKSM 12 Page 64


#MMS 33

What is the best management for a baby born to a mother who developed varicella zoster 2 days
priorto delivery? (INTAKE31)
a. Isolate the baby
b. Advice against breast feeding
c. IV Acyclovir
d. Varicella Immunoglobulin
e. Varicella immunization
T

True about chicken pox, (INTAKE 29)


a. Not infectious during prodromal period
b. Recognized risk for Reye’s Syndrome
c. Acyclovir given for immunedeficient patients
d. Notifiable disease in Sri Lanka
F T T T

Reye’s Syndrome
Causes
 Aspirin
 After the chicken fox /influenza

Prodromal period -mild unspecific symptoms that signaling impending of a diseases

Chicken fox infectious 2 days before vesical appear to 2 days after last vesicle rupture

Treatment
 VZIG should be given 7 days within infection
 Patient with mild symptoms and bulles give acyclovir as prophylaxis

Measles
A child has presented with high fever, cough and cold for 4 days. Following admission, he developed
a maculopapular rash behind his right ear. What is the most likely diagnosis? (COL2016)
a) Infectious episcleritis
b) Measles
c) Parvo virus infection

T
Infectious episcleritis
o Localized pain, intense erythema, unilateral; blood vessels bigger than in conjunctivitis;
scleritis may cause globe perforation
o Episcleritis is self-limiting; topical steroids for fast relief
o Idiopathic autoimmune disease (e.g., SLE, Henoch-Sch ö nlein purpura)

PEDIATRICS MCQ REVIEW – LKSM 12 Page 65


#MMS 33

Viral fever /investigations


An elevated level of which of the following are specific to bacterial infection?
A. Temperature
B. Serum procalcitonin
C. Platelet count
D. Lactate dehydrogenase level
E. Toxic granules in granulocytes

F T F T

 The measurement of serum procalcitonin, a specific marker of severe bacterial infections


 Platelet count drop in viral infection
 LDH – elevate in non-Hodgkin’s lymphoma
 High temperatures can have even in dengue infection

Best measure to reduce temperature in suspected child with viral fever(INTAKE31)


A. Sponging with lukewarm water
B. Oral PCM
C. Oral Ibuprofen
D. Rectal Diclofenac Na
E. Keep ice bags on forehead
T

Kawasaki disease
Most essential feature to diagnose Kawasaki disease(INTAKE 30)
1. Fever >5 days
2. Non purulent conjunctivitis
3. Serositis
4. UL cervical lymphadenopathy
T

True regarding Kawasaki disease (COL2016)


a) Kawasaki is diagnosed by a diagnosed criterion
b) Causes purulent conjunctivitis
c) prednisolone is a treatment option
d) Peeling off the skin is a late feature
e) Viral infection is an aetiological factor

T F T T F
Kawasaki disease
• Mainly affect 6 months – 4 years (80% of cases in children < 4 yrs, 5% of cases in children >
10 yrs )
• Aetiology unknown
• Systemic vasculitis

PEDIATRICS MCQ REVIEW – LKSM 12 Page 66


#MMS 33

Criteria
2. Erythema&/or fissuring of lips and oral cavity(90%+ of cases)
3. Changes in peripheral extremities –edema (90%+ of cases)
4. Bilateral non-purulent conjunctival injection(90%+ of cases)
5. Polymorphous rash(95%+ of cases)
6. Cervical lymphadenopathy(~75% of cases)

TX
• IVIG and aspirin
• Other drugs for reduce inflammation
o Corticosteroids
o Infliximab
o Cyclosporin

Infectious mononucleosis
3-year-old baby presented with fever for 9 days, cervical lymphadenopathy and inguinal
lymphadenopathy. No supratrochlear lymphadenopathy. Petechiae on soft palate and throat
inflamed. Mild hepatomegaly of 2 cm and soft splenomegaly. What is the most probable diagnosis?
(COL2016)
a) Infectious mononucleosis
b) Lyme disease
c) Miliary TB
d) Typhoid fever

Lyme disease
 Caused by the spirochaete B. burgdorferi
 Classical skin lesion known as erythema migrans, a painless red expanding lesion with a
bright red outer spreading edge.
 Often accompanied by fever, headache, malaise, myalgia, arthralgia, and lymphadenopathy.
 These features fluctuate over several weeks and then resolve.
 Dissemination of infection in the early stages is rare, but may lead to cranial nerve palsies,
meningitis, arthritis, or carditis.
 The late stage of Lyme disease occurs after weeks to months with neurological, cardiac, and
joint manifestations. Neurological disease includes meningoencephalitis and cranial
(particularly facial nerve) and peripheral neuropathies. Cardiac disease includes myocarditis
and heart block. Joint disease occurs in about 50% of cases and varies from brief migratory
arthralgia to acute asymmetric monoarthritis and oligoarthritis of the large joints. Recurrent
attacks of arthritis are common. In 10% of cases, chronic erosive joint disease occurs months
to years after the initial attack.

PEDIATRICS MCQ REVIEW – LKSM 12 Page 67


#MMS 33

Infectious mononucleosis
o Exudative tonsillitis
o Petechiae at junction of hard and soft palate
o Cervical (especially posterior cervical), axillary and inguinal lymph nodes(discrete
non tender)
o Splenomegaly 50% ,
o hepatomegaly 15%
o Jaundice rare 5%
o Pruritic maculopapular rash 90% after antibiotics (penicillin or amoxicillingroup )

Worm infections
A 6 yrs old boy presents with diarrhea and confirmed of having enterovirus vermicular infection
What is true about next step of management? (INTAKE 30)
1. He should be treated with mebendasole and pyrantal pamoate
2. All family members should be treated with mebendasole with same dose
T

Mebendazole
* Good for children
* Contraindicated in less than 1 year
* Same dose for adult and children 100 mg /BD /3 days

Albendazole
* Contraindicated in less than 1 year
* 400 mg adult dose /single
* >1 year 200 mg dose /single

Enterobios vermicularis
 Mebendazole 100 mg single dose and repat in 7 days

Abscess
Characteristic features of retropharyngeal abscess(INTAKE32)
a. Torticollis
b. Hoarseness
c. Neck stiffness
d. Dysphagia
e. Stridor

Fever, irritability, decreased oral intake, and drooling. Neck stiffness, torticollis, and refusal to move

PEDIATRICS MCQ REVIEW – LKSM 12 Page 68


#MMS 33

the neck may also be present. The verbal child might complain of sore throat and neck pain. Other
signs can include muffled voice, stridor, respiratory distress, or even obstructive sleep apnea.

Treatment options include intravenous antibiotics with or without surgical drainage. A 3rd-
generation cephalosporin combined with ampicillin-sulbactam or clindamycin to provide anaerobic
coverage is effective.
Most commonly in children < 3-4 yr of age, with boys affected more often than girls.

Skin infections
Pruritus significant in(INTAKE32)
a. Eczema
b. Urticaria
c. Measles
d. Enterobiasis
e. HSP

T T T F

Anti-biotics
Macrolides are indicated for the treatment of(INTAKE32)
a. Mycoplasma pneumonia
b. Pertussis
c. Chlamydial conjunctivitis
d. Leptospirosis
e. Strep pneumonia

T T T

Vaccines and eradication


Regarding the immunization programme in Sri Lanka, (COL2016)
a) Rubella vaccine not indicated when child already got the disease.
b) OPV is given at 5 years.
c) When a vaccine lapses by 3 months the schedule should start from the beginning.
d) IPV is a live attenuated vaccine
e) JE is given at 12 months.

T T F F T

• Any live vaccine C/I in pregnancy


• Not a contraindicated anything rather than previous history of reaction
• Vaccine lapses – give the miss dose only at that day then Live vaccine apart prom 1 month
(IPV vccine can give other hand that day even )

PEDIATRICS MCQ REVIEW – LKSM 12 Page 69


#MMS 33

• Dose miss? no need to give 1st dose again start with missing dose

Live vaccines
• BCG
• OPV
• MMR
• JE
• ROTA

Preventable diseases by active or passive immunization in neonatal period, (INTAKE31)


A. Congenital rubella.
B. Tetanus.
C. Meningitis.
D. Pertussis.
E. TB.
T

Neonatal period is up to 28 days


BCG is the only vaccine given in neonatal period

Birth BCG
2,4,6, months DPT OPV Hep B
9 months Measles, [Vit A]
18 months DPT OPV [Vit A]
3 yrs MR [Vit A]
5 yrs DT OPV
10-14 yrs Rubella, aTd

EPI schedule in Sri Lanka(INTAKE32)


a. MMR vaccine given at 9months and 3years
b. BCG vaccine is given prior to discharge in preterm babies regardless of their gestational age
c. Febrile convulsion is a contraindication for DTP vaccine
d. Pertussis is a whole cell vaccine
e. HPV is given for both male and female

T T T F F
Febrile convulsion is a contraindication for JE vaccine

Live vaccines should not be administered within? (COL2021)


a. One month of administering another live vaccine
b. 3 months of blood transfusion
c. One month after long term high dose steroids
d. 3 months after stopping immunosuppressive therapy
e. One month of IVIG

T F F F T

PEDIATRICS MCQ REVIEW – LKSM 12 Page 70


#MMS 33

Live vaccine Avoid


× Reticular endothelial systemic malignancies
× In 1st 3 month of pregnancy
× In immunocompromised

Live Vaccine with steroids relation


 Low dose steroid can give vaccine after stopping drug
 High dose steroid can give vaccine after 1 month stopping drug
 Patient with steroid replacement therapy can give the vaccine
 Topical steroid no problem

Should not give with Anaphylaxis but mild allergic reaction can give with under observation
Live Vaccine postpone
× Acute febrile illness >100 F
× Live vaccine given within less than one month
× If blood product /IG given within 3 moths
× IF petit get chemotherapy within 3 months

NO specific gap between


 Oral live vaccine
 Inactivated vaccine
 Live and inactivated vaccine

*multiple inactivated vaccines can give in same day for different place

First communicable disease to be eradicated from sri Lanka. (INTAKE32)


a. Poliomyelitis
b. Measles
c. Neonatal tetanus
d. Diphtheria
e. None of the above

Diseases which have been virtually (almost) eliminated from Sri Lanka by effective public health
interventions include, (INTAKE31)
A. Poliomyelitis
B. Endemic cretinism.
C. Filariasis.
D. Myelomeningocele.
E. Malaria.
T T T

Elimination – NOT at this country


Eradication – NOT in world

Eliminated disease

PEDIATRICS MCQ REVIEW – LKSM 12 Page 71


#MMS 33

 Poliomyelitis – 1993
 Malaria -2012
 Filariasis – 2016
 Measles -2019
 Diphtheria -2019

Eradicated disease
 Small fox – 1977

Which of the following is not giving by ID route(INTAKE32)


a. Mantoux
b. Penicillin sensitivity test
c. BCG vaccine
d. Allergic skin test
e. DTP

IPV and BCG – ID


DTP - IM

Immunodeficiency
Most common cause of 2ry immunodeficiency(INTAKE 30)
1. HIV
2. Malnutrition
3. Measles
4. Steroid
5. Cytotoxic
T

Immunodeficiency least/not caused by, (INTAKE 29)


A. Measles
B. Protein energy malnutrition
C. Prolonged bronchodilator therapy
D. Overwhelming infections
T

Immune deficiencies
 primary immune deficiencies
 2ry immune deficiencies (more common)
Caused by
 intercurrent bacterial or viral infection
 malignancy
 malnutrition
 Measles /HIV infection
 immunosuppressive therapy
 splenectomy or nephrotic syndrome.

PEDIATRICS MCQ REVIEW – LKSM 12 Page 72


#MMS 33

RESPIRATORY

Stridor
8 month old baby presented with sudden onset stridor. With a past history of cough and cold fever for 2
day duration. On admission respiratory rate 140, BP 90/60. Oxygen saturation 90%.on auscultation
bilateral rhonchi and inspiratory crept. Next appropriate management(INTAKE 30)
1. nebulize with salbutamol with iv hydrocortisone
2. nebulize with salbutamol with iv budasonide
3. nebulize adrenaline with O2
4. nebulize adrenaline with iv hydrocortisone
5. iv hydrocortisone with O2
T

viral croup – caused by parainfluenza


preceded by fever or acute coryza

Mild Saturation normal Dexamethasone 0.15 mg/kg oral


Moderate RR increase Spo2 normal Dexamethasone 0.6 mg /kg oral
Nebulize with budesonide
Sever Spo2 < 93 % Nebulize with adrenaline

Best management for viral croup is, (INTAKE 29)


A. IV Hydrocortisone
B. IV Dexamethasone
C. Nebulize with Salbutamol
D. Nebulize with Adrenalin
E. Nebulize with Budesonide
T

Oral dexamethasone

18-month-old baby presented with a 2 days’ history of coryzal symptoms, fever & cough. On
examination found to have a stridor. What is the immediate management option? (COL2016)
a) Nebulize with adrenalin
b) Nebulize with Budesonide
c) Nebulize with Ipratropium Bromide
d) Humidified oxygen
e) Nebulize with Salbutamol

PEDIATRICS MCQ REVIEW – LKSM 12 Page 73


#MMS 33

• Mild croup – dexamethasone 0.15mg/kg orally


• Moderate croup – dexamethasone 0.6mg/kg orally OR nebulize with budesonide (If oral route
not available)
• Sever croup – nebulize adrenalin 0.5ml/kg 1:1000 solution (not with congenital heart disease
patent )

Bronchiolitis
A 6-month-old child presented with rhinorrhoea for 4 days, cough for 2days, and difficulty in breathing
for 1 day. On examination respiratory rate is 50/min, mild intercostal and subcostal recessions were
present. Auscultation revealed vesicular breathing with prolonged expiration. There were rhonchi and
bilateral basal crepitations as well. No fever and the child is otherwise well. Most likely organism to
cause this? (COL2021)
a. Adenovirus
b. Corona virus
c. Metapneumovirus
d. Influenza virus
e. Respiratory syncytial virus

Bronchiolitis
× Bronchiolitis is the commonest serious respiratory infection of infancy
× 90% are aged 1–9 months (bronchiolitis is rare after 1 year of age).
× Respiratory syncytial virus (RSV) is the pathogen in 80% of cases.
× The remainder are accounted for by human metapneumovirus, parainfluenza virus, rhinovirus,
adenovirus, influenza virus, and Mycoplasma pneumoniae.

PEDIATRICS MCQ REVIEW – LKSM 12 Page 74


#MMS 33

× Dual infection with RSV and human metapneumovirus is associated with severe bronchiolitis.

Clinical picture
 Sharp, dry cough
 Tachypnoea
 Subcostal and intercostal recession
 Hyperinflation of the chest: – Prominent sternum – Liver displaced downwards
 Fine end-inspiratory crackles
 High-pitched wheezes – expiratory > inspiratory
 Tachycardia
 Cyanosis or pallor.

Whooping cough
A 7-year-old girl presented with spasmodic bouts of cough followed by a whoop for 10 days. She was
diagnosed with a whooping cough and was started on erythromycin. What is the most important reason
for starting erythromycin? (COL2021)
a. Minimize disease complications
b. Reduce disease transmission
c. Reduce duration of hospital stay
d. Reduce the severity of symptoms
e. Reduce the duration of symptoms

T
Whooping cough
 Highly contagious
 Caused by Bordetella pertussis
 3 phases
o week of coryza (catarrhal phase)
o characteristic paroxysmal or spasmodic cough followed by a characteristic inspiratory
whoop (paroxysmal phase)
o The symptoms gradually decrease (convalescent phase)

 apnoea rather than whoop, which is potentially dangerous in infants


 Complications of pertussis, such as pneumonia, convulsions and bronchiectasis, are uncommon,
but there is still a significant mortality, particularly in infants
 Diagnosis: culture of organism on per-nasal swab, marked lymphocytosis on blood film.
 Although erythromycin eradicates the organism, it decreases symptoms only if started during
the catarrhal phase
 close contacts should receive erythromycin prophylaxis
 Immunisation reduces the risk of developing pertussis and the severity of disease in those
affected, but does not guarantee protection.

PEDIATRICS MCQ REVIEW – LKSM 12 Page 75


#MMS 33

Asthma
A 3-year-old girl with well-controlled asthma is brought with a history of sudden onset of difficulty in
breathing while playing with her sister. There is no history of fever or cough. On examination, RR 60/min
with recessions, SPO2 85%. She showed poor response to 3 cycles of back-to-back nebulization. What is
the most appropriate next step of management? (COL2021)
a. Arrange an ICU bed
b. Arrange an urgent chest X-ray
c. Urgent ENT referral
d. Commence IV cefotaxime
e. Perform an ABG

T
Life threating asthma
 Cyanosis
 Bradycardia
 Silent chest
 <85 %

7 year old girl presented to the OPD with acute onset wheezing. She was nebulized twice and then
admitted to the ward. O/A RR 50/min, PR 110/min, air entry equal with vesicular breathing and
prolonged expiration B/L What is the next step in Mx(INTAKE32)
a. MgSO4
b. Aminophylline
c. High flow O2 via face mask
d. Nebulize with Ipratropium
e. Repeat salbutamol nebulization

T
Acute asthma
 Oxygen driven nebulization – 3 times /Repeat every 20 minutes
 Oral prednisolone 2 mg/ kg / Intravenous hydrocortisone 4 mg/kg
 Call for seniors
 2nd line - Magnesium sulphate bolus therapy
 3rd line - IV aminophylline bolus / IV salbutamol

A 6 year old boy admitted with acute severe asthma. For the last month he has had two nebulization at
OPD visits. He is on beclamethasone 400 mcg bd. He has frequent night symptoms. What would you
give him in discharge? (COL2016)
a) Beclamethasone 800mcg with spacer
b) Beclamethasone 800mcg with spacer and mask
c) Oral prednisone
d) Salmeterol and fluticasone MDI with spacer
e) Beclamethasone DPI

PEDIATRICS MCQ REVIEW – LKSM 12 Page 76


#MMS 33

All patient use spacer under 12 years


o < 4 year – spacer with mask (Baby haler -Can’t use without mask )
o >5 years – spacer

PEDIATRICS MCQ REVIEW – LKSM 12 Page 77


#MMS 33

Bronchiectasis
A 4-year-old girl is being investigated for bronchiectasis. Which of the following is of significance to
etiology? (COL2021)
A. Recurrent otitis media
B. Chronic diarrhea with weight loss
C. Delayed passage of meconium
D. Maternal IV drug use
E. Long term corticosteroids use

T T T • Failure to thrive
Bronchiectasis • Recurrent chest infections
 Generalised bronchiectasis causes • Malabsorption, steatorrhoea
o cystic fibrosis Young child
o primary ciliary dyskinesia • Bronchiectasis
o immunodeficiency • Rectal prolapse
o chronic aspiration • Nasal polyp
 Focal bronchiectasis causes • Sinusitis / OM
o previous severe pneumonia Older child and adolescent
o congenital lung abnormality • Allergic bronchopulmonary
o obstruction by a foreign body aspergillosis (ABPA)
• Diabetes mellitus
Clinical features of cystic fibrosis • Cirrhosis and portal
Infancy hypertension
• Meconium ileus in new-born
• Distal intestinal obstruction
period - intestinal obstruction
(DIOS, meconiumileus
with vomiting, abdominal
equivalent)
distension and failure to pass
• Pneumothorax or recurrent
meconium in the first few days
haemoptysis
of life
• Sterility in males
• Prolonged neonatal jaundice

PEDIATRICS MCQ REVIEW – LKSM 12 Page 78


#MMS 33

Pneumonia
A 3 year old boy was treated for R/S lower lobar pneumonia 2 weeks back. Now he presented with low
grade fever and productive cough for 3 days. On chest xray findings were same as last time with patchy
R/ lower lobe consolidation. Mother also states a history of choking and coughing whilefeeding“chick
pea” (kadala) for child before his initial symptoms. What is the best investigation to diagnose is
condition? (INTAKE31)
A. Mantoux test
B. Sputum culture with ABST
C. Bronchoscopy
D. Immunoglobulin assay
E. Blood culture
T

6 year old boy admitted with low grade fever and cough for 2 days duration. Cough was dry and
paroxysmal and associated with wheezing. Also complained of headache and body aches. CXR – B/L
Hilar shadows, WBC –7600, N% - 54, ESR – 30, CRP – 12. What is the most likely aetiological factor?
(COL2016)
a) Heamophilus influenzae
b) Mycobacterium tuberculosis
c) Mycoplasma pneumoniae
d) Pneumocystis jirovecii
e) Streptococcus pneumoniae

 Mycoplasma pneumoniae
o Radiographic findings are not specific.
o Pneumonia is usually described as interstitial or bronchopneumonic, and involvement is
most common in the lower lobes, with unilateral, centrally dense infiltrates present in
75% of cases.
o The white blood cell and differential counts are usually normal, whereas the erythrocyte
sedimentation rate is often elevated.
o It has extra pulmonary manifestations
 Pneumocystis jirovecii pneumonia (interstitial plasma cell pneumonitis)
o immunocompromised person is a life-threatening infection.
 Heamophilus influenzae
o Should have pre existing lung disease

PEDIATRICS MCQ REVIEW – LKSM 12 Page 79


#MMS 33

6-month-old child who was diagnosed to have TOF presented with dyspnoea, wheezing, rhinorrhoea.
CXR shows B/L patchy consolidations. He was not improved despite treatment with IV Cefotaxime and
IV Gentamicin. Next best treatment, (COL2016)
a) Intubate and ventilate
b) IV Morphine
c) Sputum culture
d) Treat with Oseltamivir
e) Urgent BT shunt

 Influenza types A and B cause predominantly respiratory illness. The onset of illness is abrupt
and is dominated by fever, myalgias, chills, headache, malaise, and anorexia; coryza, pharyngitis,
and dry cough are associated features overshadowed by the other systemic signs
 The predominant symptoms may localize anywhere in the respiratory tract, producing an
isolated upper respiratory tract illness, croup, bronchiolitis, or pneumonia.

A chest X-ray may confirm the diagnosis but cannot reliably differentiate between bacterial and viral
pneumonia. In younger children, a nasopharyngeal aspirate may identify viral causes, but blood tests,
including full blood count and acute-phase reactants are generally unhelpful in differentiating between a
viral and bacterial cause
Causes for non-resolving pneumonia
 complications, such as empyema;
 bacterial resistance;
 nonbacterial aetiologies such as viruses and aspiration of foreign bodies or food;
 bronchial obstruction from endobronchial lesions, foreign body, or mucous plugs;
 pre-existing diseases such as immunodeficiencies, ciliary dyskinesia, cystic fibrosis,
pulmonary sequestration, or cystic adenomatoid malformation;
 other non-infectious causes (including bronchiolitis obliterans)

3-year-old previously treated for right lower lobe pneumonia 3 months back. X-ray not resolved. Now
complaints of persistent cough. What is the best investigation for the diagnosis? (COL2016)
a) 24hr PH
b) Sputum culture & ABST
c) Monteux
d) Immunoglobulin testing
e) Sweat chloride

T
Non resolving pneumonia – do first culture and ABST then bronchoscopy

PEDIATRICS MCQ REVIEW – LKSM 12 Page 80


#MMS 33

Tonsillitis
10 year old boy presented with a follicular tonsilitis. He has a past history of urticarial rash after eating
pineapple. Best drug to be used in the primary care setting(INTAKE31)
A. Oral penicillin
B. Amoxicillin
C. Clarythromycin
D. Erythromycin
E. Ciprofloxacin
T

 It is usually due to viral infection (mostly adenoviruses, enteroviruses, as well as rhinoviruses).


In the older child, group A β-haemolytic streptococcus is a common pathogen.
 Antibiotics (e.g. penicillin V or erythromycin if there is penicillin allergy) are often prescribed for
severe pharyngitis and tonsillitis even though only a third are caused by bacteria

Best Indication for adenotonsilectomy (INTAKE 30)


1. Recurrent serious otitis media
2. Failure to thrive

Absolute indication
 Sleep apnea
 Chronic respiratory tract obstruction
 Malignancy

Relative
 Recurrent/chronic tonsillitis
 Diphtheria careers

Adenectomy
Indication – prolong serous otitis media or recurrent OM

Bronchitis
Regarding bronchitis, (INTAKE31)
A. More prominent in 2nd half of life.
B. Responds to bronchodilators.
C. Humidified oxygen is effective in treatment.
D. Bronchial breathing can be heard.
E. Permanent damage to bronchial tree can occur.

In acute bronchitis in children, cough with sputum and fever are the main symptoms.

PEDIATRICS MCQ REVIEW – LKSM 12 Page 81


#MMS 33

Auscultation of the chest may be unremarkable at this early phase. As the syndrome progresses and
cough worsens, breath sounds become coarse, with coarse and fine crackles and scattered high-pitched
wheezing.
The cough may persist for about 2 weeks, or longer with pertussis or Mycoplasma infections.
There is no evidence that antibiotics, cough suppressants or expectorants speed recovery.

Pneumothorax
Conditions, which can shift the trachea away from the effected side, in 1 year old child, (INTAKE31)
A. Lung collapse.
B. Tension pneumothorax.
C. Large pleural effusion.
D. Mediastinal tumor
E. Complete lobar pneumonia.
F T T T F

NHL common at adolescence

2 year old boy was diagnosed with Asthma. He had acute exacerbation and was treated at home with
Salbutamol inhaler. But there was no adequate response. On admission breath sounds on the R/S chest
was reduced and resonance increased on percussion. What is the best management option at hospital?
(INTAKE 29)

A. 100% O2 via face mask


B. Nebulize with Salbutamol alone
C. Nebulize with Salbutamol and Iptratropium
D. Right sided needle thoracentesis
E. Inset a IC tube with water seal
T

Tension pneumothorax – trachea is deviated


Mx –needle thoracosentases

Pneumothorax – trachea not deviated


Mx – IC tube with water seal after confirming with chest XRAY

PEDIATRICS MCQ REVIEW – LKSM 12 Page 82


#MMS 33

CARDIAC

Sudden cardiac death


Sudden death in infants, (INTAKE31)
A. HOCM.
B. Long PR interval.
C. AS
D. MR.
E. Pericarditis.
F F T

Causes for sudden death in infants


 Sudden infant death syndrome
 Medium-chain acyl-coenzyme A dehydrogenase deficiency (MCAD deficiency)
 Infant botulism
 Long QT syndrome (accounting for less than 2% of cases)
 Helicobacter pylori bacterial infections
 Shaken baby syndrome and other forms of child abuse
 Overlaying, child smothering during carer's sleep

Sudden death in an adolescent can be due to(INTAKE 30)


1. Hypertrophic obstructive cardiomyopathy
2. AV malformation in brain
3. Anaphylaxis
4. PCM poisoning
5. Long QT syndrome
T T T F T

Heart failure
A 6-week old baby boy presented with difficulty in feeding and dyspnoea. Due to feeding difficulty, he
was fed with expressed breast milk during the past week. On examination, he was found to have cold
clammy peripheries. PR 170bpm and bibasal crepitation were present. The liver was palpable 3cm
below the costal margin and a systolic murmur was heard on the lower sternal edge. What is the next
step in management? (COL2021)
a. Administer intravenous 0.9% NaCl bolus
b. Administer IV Furosemide
c. Commence IV Cefotaxime
d. Start intravenous prostaglandin infusion
e. Nebulise with 3 percent normal saline

PEDIATRICS MCQ REVIEW – LKSM 12 Page 83


#MMS 33

Heart failure
 Symptoms
o Breathlessness (particularly on feeding or exertion)
o Sweating
o Poor feeding
o Recurrent chest infections.
 Signs
o Poor weight gain or ‘faltering growth’
o Tachypnoea
o Tachycardia
o Heart murmur, gallop rhythm
o Enlarged heart
o Hepatomegaly
o Cool peripheries

10M old child was presented with difficulty in breathing and inconsolable cry for 1 day. O/E he has a
pan-systolic murmur at left lower sternal edge and tender hepatomegaly. Immediate management for
relive his symptoms, (INTAKE32)
a. O2 via face mask
b. IV frusemide
c. IV propranolol
d. 0.9% N/S bolus
e. IV Digoxin

Arrhythmias
A 10-month-old baby presented with episodic crying. On examination peripheries were warm, CRFT was
2 seconds. Heart rate was 230 beats per minute. ECG was taken and it showed sinus rhythm but no P
waves were identified. What is the next management option? (COL2021)
a. Administer intravenous 0.9% NaCl bolus
b. Administer intravenous adenosine
c. Compression of the face with ice water-filled glove
d. Apply pressure on orbit
e. Synchronized DC shock

PEDIATRICS MCQ REVIEW – LKSM 12 Page 84


#MMS 33

 A sinus rhythm origin from sinus node


 Characterised by the presence of correctly oriented P waves on the electrocardiogram (ECG)

SVT
 SVT Is the most common
 Rapid Heart Rate
o >220 (In a younger than 2 year old)
o >170 (In a elder than 2 year old)
 A regular narrow QRS tachycardia without easily seen P waves
 If unstable
o Synchronized DC shock
 If stable (asymptomatic with good perfusion and also those who are in mild heart failure )
o Vagal manoeuvres may be tried- Ice water applied to the face - PREFERRED
o Medications are indicated if these manoeuvres are unsuccessful- Adenosine ( treatment
of choice )

Congenital heart defects


5 days old baby who was discharged on day 2 after NVD. On the 5th day, she presented with shortnessof
breath and cyanosis. What is the immediate management step? (INTAKE 29)
A. High flow Oxygen
B. 10 % Dextrose IV
C. IV Hydrocortisone
D. IV Prostaglandine
E. 0.9% NaCl bolus
T

2 day old neonate was brought to the emergency department with shortness of breath and cyanosis.
Examination of the baby revealed a heart rate of 140 ,of 70 and SpO2 of 94. The blood pressure
measurement of upper limb was 80/60 and lower limb was 50/30 (not sure of values )no other
abnormalities . What is the first step in the management of this child(INTAKE32)
a. IV 0.9% normal saline bolus
b. IV prostaglandin infusion
c. O2 via face mask
d. Cardiac referral for surgery

T
Management of Outflow obstruction in the sick infant
• resuscitate (ABC)
• prostaglandin should be commenced at the earliest opportunity
• referral is made to a cardiac centre for early surgical intervention.

PEDIATRICS MCQ REVIEW – LKSM 12 Page 85


#MMS 33

5 year old girl with moderate secundum ASD with no evidence of complications, what is the next
appropriate management? (INTAKE31)
A. Surgery before starting school
B. Surgery before marriage
C. Transcatheral closure of the ASD
D. Reassure that it will close spontaneously within 2 -3 years
T

Secondum ASD - Catheter device closure at 3–5 years of age


Primum ASD - Surgery at 3 years of age

5 year old boy was diagnosed to have a short systolic murmur at left upper sternal edge, which
changed in intensity when changing posture. Second heart sound was normal. No carotid thrill or
radiation. What is the most likely cause? (COL2016)
a) Pulmonary stenosis
b) ASD
c) Innocent murmur
d) Tetralogy of Fallot
e) Aortic stenosis

 The most common innocent murmur is a medium-pitched, vibratory or “ musical, ”relatively


short systolic ejection murmur, which is heard best along the left lower and midsternal border
and has no significant radiation to the apex, base, or back. It is heard most frequently in
children between 3 and 7 yr of age. The intensity of the murmur often changes with respiration
and position and may be attenuated in the sitting or prone position
 ASD – ejection systolic murmur at upper sternal edge / fixed and wide split of 2nd heart sound
 TOF – loud hash ejection systolic murmur at left sternal edge
 Small VSD – loud pan systolic murmur at lower sternal edge / Quiet P2
 Large VSD – apical mid diastolic murmur /loud P2

Aortic stenosis (COL2016)


a) Presenting with chest pain in children
b) Carotid thrill present
c) Ejection systolic murmur best heard at upper left sternal edge
d) Causes biventricular hypertrophy
e) Second heart sound is loud

PEDIATRICS MCQ REVIEW – LKSM 12 Page 86


#MMS 33

T T F F F

AS
Presentation
 asymptomatic
 sever stenosis – HF

**Bi ventricular hypertrophy is a feature of large VSD

Which one of the following is not correctly matched? (INTAKE31)


A. Jugular venous hump – anaemia
B. Continuous murmur – PDA
C. Mid diastolic murmur – mitral stenosis
D. Fixed splitting of S2 – ASD
E. Pericardial rub – Pericarditis
T

PEDIATRICS MCQ REVIEW – LKSM 12 Page 87


#MMS 33

Regarding L to R shunt(INTAKE 30)


1. ASD
2. VSD
3. PDA
4. TOF
5. Transportation of great vessels
T T T F F

Left to right shunt seen in, (INTAKE 29)


A. ASD
B. VSD
C. TOF
D. TGA
E. PDA

T T F F T

PEDIATRICS MCQ REVIEW – LKSM 12 Page 88


#MMS 33

KUB AND GENITAL

Scrotal problems
A 9-year-old boy presents with a history of swelling and pain in the scrotum for 1 day. He has been
treated for a urinary tract infection 5 days back. On examination, there is redness, tenderness, and
swelling over the right hemiscrotum. The pain is relieved by elevation of the scrotum. What is the most
likely cause for this presentation? (COL2021)
a. Acute epididymitis
b. Idiopathic scrotal edema
c. Incarcerated hernia
d. Torsion of the testicular appendage
e. Testicular torsion

T
Testicular torsion Presentation
 Usually 10-25yrs
 Sudden onset severe pain in one testis
 May have lower abdominal pain (testis supplied by T10)
 Assoc. c¯ n/v
 May be Hx of previous testicular pain or torsion

UTI
WOTF are T/F Regarding investigations done in urinary tract infection in a child (INTAKE32)
a. MCUG done in all children below the age of 5
b. DTPA done in PUJ
c. DMSA done after 6 weeks
d. Xray KUB is indicated in all boys following UTI
e. USS done within 48 hrs in suspected pyelonephritis

F T F F T
UTI radiological investigations
 USS KUB
o Acute phase – any age atypical UTI and recurrent UTI only < 6 months age
o With in 6 weeks – any age all types of UTI (afebrile,simple febrile,atypical,recurremt )

 DMSA
o Any age with Atypical and recurrent UTI
o After 6 months

PEDIATRICS MCQ REVIEW – LKSM 12 Page 89


#MMS 33

 MCUG
o Only <6 months age atypical and recurrent UTI
o After ^ weeks

 DTPA
o PUJ obstruction

Which is the commonest cause for a recurrent uti in a three year old child(INTAKE32)
a. Posterior urethral valve
b. PUJ obstruction
c. VUR
d. Labial adhesions
e. Renal Calculi

T
Assessment of Risk factors
 Faecal mass (constipation)- DRE without scaring/exciting children. Take consent from parent.
 Labial adhesions
 Long Tight prepuce/phimosis
 Palpable bladder
 Ballotable kidneys
 Evidence of spinal lesion (lump, tuft of hair, asymmetry, sinus)
 Examine the lower limbs for evidence of spinal lesion(gait, power, tone and reflexes)
 Assess or question the urine flow
 Assess for constipation and persistent LUTS

A 3 months old baby boy, presented with high-grade fever and straining while micturition for 2 days.
He has had a poor stream since birth. On examination, he was febrile and had a palpable mass in the
suprapubic region. What is the next step in management? (COL2021)
a. Urgent MCUG
b. Arrange USS of the abdomen
c. Catheterization
d. Take urine for culture and start empirical antibiotics
e. Urgent surgical referral

T
MCUG is the diagnostic test for PUV
 Before the procedure confirm negative urine culture
 Therapeutic antibiotics for 3-5 days within procedure

PEDIATRICS MCQ REVIEW – LKSM 12 Page 90


#MMS 33

2 months old child presented with high grade fever >39ºC for 2 days. He was drowsy and feeding was
poor. What is the important investigation for the management? (INTAKE32)
a. FBC
b. CRP
c. Urine culture
d. Lumbar puncture
e. Blood culture

8 month old boy was investigated for febrile UTI. In his USS abdomen dilated right pelvis was seenand
PUJ obstruction suspected. Which of the following investigation confirm the diagnosis? (INTAKE31)
A. Plain xray gut
B. CT scan
C. DMSA
D. DTPA
E. MCUG
T

2 months old child presented with high grade fever >39ºC for 2 days. He was drowsy and feeding was
poor. What is the important investigation for the management? (INTAKE 29)
A. FBC
B. CRP
C. Urine culture
D. Lumbar puncture
E. Blood culture
T

Baby boy investigated with USS showing R/S dilated renal pelvis, suggesting of PUJ obstruction. What is
the best investigation to confirm the diagnosis?
a) CT scan
b) DMSA
c) DTPA
d) IV pyelogram
e) MCUG

PEDIATRICS MCQ REVIEW – LKSM 12 Page 91


#MMS 33

PUV
You are asked to see a baby boy aged 50 hrs. who has not passed urine since birth. Baby is active, looks
well, feeding normally. Late antenatal ultrasound scan had detected oligohydramnios. Next step inthe
management(INTAKE31)
A. Reassure mother and ask her to observe the baby for further 24 hrs.
B. Top up with formula feed
C. Catheterize and check whether urine draining
D. USS KUB
E. Normal saline bolus
T

50-hours old baby has not passed urine since birth. Late antenatal scan showed oligohydramnios. Baby
is active and well. What is the next step of management? (COL2016)
a) IV N/S bolus
b) USS
c) Palpate for bladder
d) catheterize and check whether urine is coming
e) DMSA

• Bladder palpable – catheterize


• If baby Active well no bladder palpable– UUS – then catheterize

VUR
VUR, (INTAKE31)
A. Causes scarring.
B. Has a familial tendency.
C. Can diagnose by USS.
D. Always corrected surgically.
E. Cause young HTN in adults.
T T F F T

 Vesicoureteric reflux (VUR) is a developmental anomaly of the vesicoureteric junctions.


 If scarring is bilateral and severe, chronic renal failure may develop. The risk for hypertension in
childhood or early adult life
 It is familial, with a 30–50% chance of occurring in first-degree relatives. It may also occur with
bladder pathology, e.g. a neuropathic bladder or urethral obstruction, or temporarily after a
UTI.
 intrarenal reflux is associated with a particularly high risk of renal scarring if UTIs occur

PEDIATRICS MCQ REVIEW – LKSM 12 Page 92


#MMS 33

 Reflux tends to resolve with age especially lower grades of VUR. High grade VUR need surgical
correction up to that antibiotic prophylaxis for UTI

Nephritic syndrome
In post-streptococcal glomerulonephritis, (COL2021)
A. An increase in hydrostatic pressure is the reason for hypertension
B. Complement 3 (C3) levels are increased during the acute illness
C. Type 3 hypersensitivity is the cause of glomerular damage
D. Angiotensin receptor blockers are the 1st line antihypertensive
E. A favourable prognosis is seen in the majority

T T T F T

Hypersensitivity type
Type 1 Ig E mediated Anaphylaxis ,asthma
Type 2 anti-body Haemolytic anaemia
mediated IgG or /intestinal nephrites /
IgM antibodies thrombocytopenia
Type 3 immune complex Serum reaction ,SLE /HSP
mediated /Arthus reaction /RA /PSGN
[SHARP]
Type 4 deleyed type Contact dermatitis /
transplant rejection

1st line for HTN – CCB ( nifidipin )


Pathophysiology of nephritic syndrome

PEDIATRICS MCQ REVIEW – LKSM 12 Page 93


#MMS 33

7-year-old boy presented with 2-day history of painless haematuria. Mother complains that the child
passes uniformly red urine with reduced output. He also complains of headache. He had a UTI 6 months
back. What is the most appropriate next investigation? (COL2016)
a) Check urine for deposits
b) Measure her blood pressure
c) Do x-ray of abdomen
d) USS
e) Renal biopsy

T
Next management is BP

Best management option in a Child presenting with nephritic syndrome .With rapid thread
pulses low blood pressure and abdominal pain? (INTAKE 30)
1. Furosemide and spironolactone
2. Salt restriction and furosemide
3. Salt free albumin and furosemide
4. furosemide and prednisolone
5. Prednisolone 60mg2
T

Hypovolemia – 20% albumin infusion followed by furosemide

Hypovolemic shock – 4.5% albumin 10ml/kg/hr bolus OR 0.9 % normal saline bolus

In HSP, (INTAKE 30)


1. Abdominal pain is a feature
2. Bloody diarrhea is a feature
3. Bleeding time increases
4. Chronic renal failure is a complication
5. Blanching rash is there
T T F T F

Colicky abdominal pain – hematemesis .Malena


CKD and HTN are complication but very rare
Rash – non blanching /symmetrical /macular popular purpuric palpable /buttocks /extensor surface of
the arms and legs, ankle // it is the 1st feature in 50 % case

PEDIATRICS MCQ REVIEW – LKSM 12 Page 94


#MMS 33

Nephrotic syndrome
Nephrotic syndrome diagnose pt presented with proteinuria 3+ 60mg/m2 of prednisolone givenfor
14days and 40mg /m2 for 28 days.Had a Previous episode,which was treated with prednisoloneand
After 7 days of treatment she developed proteinurea. This episode proteinuria+after 5days of stopping
treatment. What best explains above ? (INTAKE32)
a. steriod Sensitive nephrotic syndrome
b. steroid Dependent nephrotic syndrome
c. Minimal change disese
d. Steroid Resistant nephrotic syndrome

T
Steroid dependent
While on EOD steroid or after 2 weeks stop steroid patient developed relapse

Steroid sensitive nephrotic syndrome (Minimal change nephrotic)


 Normal renal function test
 No HTN
 No macroscopic haematuria
 Completely resolve with prednisolone

Indications for renal biopsy in nephrotic syndrome, (COL2016)


a) Macroscopic haematuria
b) Low C3 &C4
c) Age < 2 year
d) Renal failure with normovolaemia
e) Not responding to steroid for 14 days

T T F T F

Renal biopsy indications in nephrotic syndrome


× Do not respond to steroid for 4/6 weeks or have atypical features
× <6month year or >11 year
× Elevated creatinine
× Macroscopic haematuria
× HTN
× Low C3 C4 (meaning : Membranoproliferative glomerulonephritis )
× Failure to respond 28 days
× RF not attributed to hypovolemia

PEDIATRICS MCQ REVIEW – LKSM 12 Page 95


#MMS 33

12 yr old girl with generalized edema, proteinuria (+3), blood pressure 120/100, serum albumin 1.4g/dl,
UFR -RBC 40-50 cells/hpf & 10-20 pus cells/hpf. What combination is strongly suggestiveof non-minimal
change glomerular nephritis? (INTAKE 30)
1. +3,1.5,40-50
2. 40-50,1.5,120/100
3. 12 yr old,40-50,120/100
4. severe edema ,40-50,1.5
5. severe edema ,1.5,120/100
T

nephrotic syndrome non minimal changes


2-10 years
Normal BP

Electrolyte imbalance and urine changes


Causes for hypercalciuria(INTAKE32)
a. Frusemide
b. Wilson's disease
c. Hypoparathyroidism
d. Vitamin D deficiency
e. Proximal renal tubular acidosis

T
Hypercalciuria can also accompany conditions resulting in hypercalcemia, such as hyperparathyroidism,
vitamin D intoxication, immobilization, and sarcoidosis.
Hypercalciuria may be associated with Cushing syndrome, corticosteroid therapy, tubular dysfunction
secondary to Fanconi syndrome (Wilson disease, oculocerebrorenal syndrome), Williams syndrome,
distal renal tubular acidosis, or Bartter syndrome.
Hypercalciuria may also be seen in patients with Dent disease, which is an X-linked form of
nephrolithiasis associated with hypophosphatemic rickets.

PEDIATRICS MCQ REVIEW – LKSM 12 Page 96


#MMS 33

Causes for hypernatremia includes, (INTAKE 29)


A. Adrenal insufficiency
B. Rota virus diarrhea
C. Diuresis
D. Cystic fibrosis
E. Craniogenic diabetes insipidus
F T F T

 Adrenal insufficiency – hyperkalemia / hyponatremia


 Cystic fibrosis – hyponatremia
 Rota virus diarrhea – sever watery diarrhea

× Osmotic diuretics hypernatremia


× Thiazide diuretics hyponatremia
× Loop both

Causes for hypernatremia


 Hypervolemic
o Na overload (hypertonic saline )
 Hypovolemic
o Diarrhea /dehydration
o Burns
o Sweating
o Fever

 Euvolemic
o Diabetic insupidance (central – low ADH secretion from pituitary /nephrogenic – V2
receptors of kidney doesn’t respond to ADH )
o Osmotic diuretics

Myoglobin urea is a feature of(INTAKE 30)


1. G6PD
2. hump nose viper envenomation
3. rhabdomyolysis
4. HUS
5. sea snake envenomation
F F T F T

Cause – trauma /malignant hypothermia /vascular problems / glycogen storage disorders / sea snake
bite

HUS – micro angiopathic hemolytic anemia /thrombocytopenia / AKI

PEDIATRICS MCQ REVIEW – LKSM 12 Page 97


#MMS 33

PSYCHIATRIC

Nightmares
A 3 year old boy wakes up once or twice at night during sleep and cries excessively. His growthis
normal. Examination is unremarkable. What is the most likely diagnosis is, (INTAKE31)
A. ADHD
B. Hunger
C. Nightmares
D. Temper tantrums
E. Breath holding attacks
T

Nightmares
 These are bad dreams which can be recalled by the child
 Stereotyped in content
 can be due to PTSD

A 3-year-old child is brought by parents as they are concerned about his sleep. He wakes up once or
twice in the night and starts crying excessively. He has not had any significant illnesses in the past.
Growth is age appropriate and examination is normal. Most likely cause for this behaviour is (COL2016)
a) ADHD.
b) Breath holding episode.
c) Hunger.
d) Nightmare.
e) Temper tantrums.

T
Temper tantrums are ordinary responses to frustration, especially at not being allowed to have or do
something.

Mental retardation
Commonest cause for moderate mental retardation in a 6 year old boy(INTAKE 30)
1. trisomy 21
2. fragile x syndrome
3. herado cultural mental retardation
4. hypothyroidism
5. chronic protein energy malnutrition
T

Trisomy 21 – mild to moderate /but can sever

PEDIATRICS MCQ REVIEW – LKSM 12 Page 98


#MMS 33

Fragile X – moderate to sever

Causes for mental retadatation


Organic – chromosome abnormality ( pada villi /fragile X /Trisomy 21)
Cultural familial mental redatation –

T/F regarding intelligence quotient (IQ), (COL2016)


a) Determined by an intelligence test.
b) Mean IQ has changed over time.
c) IQ is an objective measurement.
d) Majority have an IQ between 85 – 115.
e) A person with IQ of 140 considered as a genius.

T F T T T

IQ range
• Mild 50 -69
• Moderate 35 -49
• Sever 20 -34
• Profound <35

Nocturnal enuresis
Which one is false regarding Nocturnal enuresis, (INTAKE 29)
A. When child reaches age of 5, 6% of children not having dry bed
B. 1st generation relatives have same condition
C. Majority associated with physical disorders
D. Associated with genetically delay in acquiring sphincter control
E. Ratio is 2:1 in male and female
T T F T T

Nocturnal enuresis
 About 6% of 5-year-olds and 3% of 10-year-olds are not dry at night
 Two-thirds of children with enuresis having an affected first-degree relative
 There is a genetically determined delay in acquiring sphincter competence
 Boys :girls 2 : 1
 Most affected children are psychologically and physically normal
 Treatment usually considered only at >6 years of age
 Management – explanation, star charts, enuresis alarm, sometimes desmopressin.

Organic causes of enuresis are uncommon but include:


 Urinary tract infection
 Faecal retention severe enough to reduce bladder volume and cause bladder neck dysfunction
Polyuria from osmotic diuresis, e.g. diabetes mellitus, or renal concentrating disorders, e.g. chronic renal
failure

PEDIATRICS MCQ REVIEW – LKSM 12 Page 99


#MMS 33

ADHD/ASD and others


In children; (COL2016)
a) Joint inattention is a feature of ADHD
b) Secondary nocturnal enuresis can be caused by family circumstances
c) Encopresis is diagnosed at 3 years
d) Separation anxiety is a cause for school refusal in a 5-year old
e) Self-injury seen in children with intellectual impairment

F T F T T

Definition of encopresis requires the voluntary or involuntary passage of feces into inappropriate places
at least once a month for 3 consecutive months once a chronologic or developmental age of 4 yr has
been reached.

Separation anxiety
• Separation anxiety is developmentally normal when it begins about 10 mo of age and tapers off
by 18 mo. By 3 yr of age, most children can accept the temporary absence of their mother or
primary caregiver.
• School refusal based on separation anxiety is typical of children under the age of about 11
years

Intellectual disability
• Intellectual disability is not treatable, many associated impairments are amenable to
intervention and therefore benefit from early identification.
• Most children with an intellectual disability do not have a behavioural or emotional disorder as
an associated impairment, but challenging behaviours (aggression, self-injury, oppositional
defiant behaviour) and mental illness (mood and anxiety disorders) occur with greater
frequency in this population than among children with typical intelligence.
• intellectual disability, often with autism.
• Autistic disorder including fragile-X, Prader-Willi, Smith-Lemli-Opitz, Rett ’ s, and Angelman ’ s
syndromes, fetal alcohol syndrome, tuberous sclerosis, neurofibromatosis, congenital rubella, or
untreated phenylketonuria

Attention deficit hyperactivity disorder


• Affects males more than females.
• Clinical features: cannot sustain attention, excessively active, socially disinhibited, easily
distracted and impulsive, poor at relationships, prone to temper tantrums, poor school
performance.
• Management: educational psychologist assessment, behavioural programmes in school,
parenting intervention, medication if necessary.

PEDIATRICS MCQ REVIEW – LKSM 12 Page 100


#MMS 33

6-year-old child presented with aggressive behaviour at school specially during reading class. But he
enjoys studying music and arts. What is the most likely diagnosis? (COL2016)
a) Mental retardation.
b) Dyslexia.
c) Refractory error.
d) Normal child.
e) Autism.

T
Autism spectrum disorder
• Presents at 2–4 years of age with impaired social interaction, speech and language disorder
and imposition of routines with ritualistic and repetitive behavior.
• Usually managed by behavior modification such as applied behavioral analysis.

Dyslexia is a disorder of reading skills disproportionate to the child’s IQ. The term is often used when
the child’s reading age is more than 2 years behind his/her chronological age. Assessment needs to
include vision and hearing and involves an educational psychologist

A 4-year-old boy is brought to you because of unruly behavior by his parents who are busy
professionals. They complain that the boy is crying incessantly and having odd behavior once picked up
from the preschool. He behaves well at pre-school and is loved by his teachers. What is the most
beneficial intervention for this child? (COL2021)
a. Behavioral therapy
b. Advice on parenting skills
c. Assess for possible ADHD
d. Extend his preschool time
e. Request the teacher to minimize his homework

PEDIATRICS MCQ REVIEW – LKSM 12 Page 101


#MMS 33

MALIGNAT

Renal tumors
2 year old presents with a mass in right lumber region. It is not ballotable but crosses to left side. What
is the most likely condition? (COL2016)
a) Hepatoblastoma
b) Neuroblastoma
c) Nephroblastoma
d) Renal cell carcinoma

Neuroblastoma
• Arise from neural crest tissue in the adrenal medulla and sympathetic nervous system.
• Clinical features
• Most children have an abdominal mass, but the primary tumour can lie anywhere along the
sympathetic chain from the neck to the pelvis.
• Classically, the abdominal primary is of adrenal origin, but at presentation the tumour mass is
often large and complex, crossing the midline and enveloping major blood vessels and lymph
nodes.
• Paravertebral tumours may invade through the adjacent intervertebral foramen and cause
spinal cord compression requiring emergency intervention to prevent devastating longterm
neurological damage.
• Over the age of 2 years, clinical symptoms are mostly from metastatic disease, particularly bone
pain, bone marrow suppression causing weight loss, and malaise
• Characteristic clinical and radiological features with raised urinary catecholamine metabolite
levels (VMA/HVA) suggest neuroblastoma.
• Confirmatory biopsy is usually obtained and evidence of metastatic disease detected with bone
marrow sampling and MIBG scan

Wilms tumour
Common presentation
• Abdominal mass
• Haematuria

Prognosis is good, with more than 80% of all patients cured.

PEDIATRICS MCQ REVIEW – LKSM 12 Page 102


#MMS 33

Bone tumors
Regarding bone tumours in children, (COL2021)
a. Is common after puberty
b. Localized pain is a feature
c. Subperiosteal new bone formation is seen in X-ray
d. Commonly metastasizes to the brain
e. Best treated with radiotherapy

T T F F F

Bone tumours
 Malignant bone tumours are uncommon before puberty. Osteogenic sarcoma is more common
than Ewing sarcoma, but Ewing sarcoma is seen more often in younger children. Both have a
male predominance.
 The limbs are the most common site. Persistent localised bone pain is the characteristic
symptom, usually preceding the detection of a mass, and is an indication for early X-ray.
 At diagnosis, most patients are otherwise well.
 A bone X-ray shows destruction and variable periosteal new bone formation
 Lung and bone marrow metastasis is common
 In both tumours, treatment involves the use of combination chemotherapy given before
surgery
 Ewing sarcoma, radiotherapy is also used in the management of local disease.

PEDIATRICS MCQ REVIEW – LKSM 12 Page 103


#MMS 33

Brain tumor
Regarding brain tumors in children(INTAKE32)
a. Commonest solid tumors in children
b. Almost always primary
c. Infratentorial are the commonest
d. Commonest solid tumour in children
e. Glioblastoma multiforme is benign

T T T T F
Astrocytoma (~40%) – varies from benign to highly malignant (glioblastoma multiforme).

PEDIATRICS MCQ REVIEW – LKSM 12 Page 104


#MMS 33

Hematological malignancies
Leukemia
ALL is least associated with, (INTAKE32)
a. Identical twins
b. Down’s Syndrome
c. First degree siblings
d. Ataxia Telangiectasia
e. Fanconi Syndrome

Increased risk of leukaemia in children (COL2016)


a) Chemotherapy
b) Down syndrome
c) Immunodeficiency
d) Fanconi anaemia

T T F T
Risk factors for leukaemia
• Radiation-This can induce genetic damage to haemopoietic precursors; ALL, AML and CML have
been seen in increased incidences in survivors of Hiroshima, Nagasaki and, more recently,
Chernobyl, and in patients treated with ionizing radiation.
• Chemicals and drugs- Exposure to benzene, used in industry, may lead to marrow damage. AML
occurs after treatment with chemotherapy drugs such as alkylating agents (e.g. melphalan) and
topoisomerase-II inhibitors (e.g. etoposide).
• Genetic factors- Fanconi anaemia, ataxia telangiectasia and Li–Fraumeni syndrome. Down’s
syndrome, monozygotic twins
• Viruses. ATLL

**Immunodeficiency is a risk factor for lymphoma

ALL is least associated with, (INTAKE 29)


A. Identical twins
B. Down’s Syndrome
C. First degree siblings
D. Ataxia Telangiectasia
E. Fanconi Syndrome

PEDIATRICS MCQ REVIEW – LKSM 12 Page 105


#MMS 33

A 10-year-old girl presented with fever and cough for 6 days. She was previously treated for ALL at 6
years of age and achieved remission. On examination, she is pale with a hepatosplenomegaly of 4cm
and is having lymphadenopathy. Hb -7.8 Wbc -48000 lymphocytes 98% blasts ++ Plt - 120. What is the
initial management? (COL2021)
a. Arrange packed red cell for transfusion
b. Commence hyper-hydration
c. Commence IV antibiotics
d. Perform bone marrow aspiration
e. Recommencement of chemotherapy

T
High dose chemotherapy is the treatment for relapses but this patient has fever ,so chemo is
contraindicated

Management
 Supportive
o Blood products
o Allopurinol
o Hickman line or Portacath
 Infections
o Gentamicin + tazocin
o Prophylaxis: e.g. co-timoxazole, ciprofloxacin
 Chemotherapy (recruited into national trials)
o Remission induction - Combination chemotherapy including steroids

PEDIATRICS MCQ REVIEW – LKSM 12 Page 106


#MMS 33

o Consolidation + CNS Rx - Cytotoxic drugs penetrate poorly into the central nervous
system / intrathecal chemotherapy
o Maintenance for 2-3yrs - Cotrimoxazole prophylaxis is given routinely to prevent
Pneumocystis pneumonia
 BMT
o Best option for younger adults

Neutropenic Sepsis
General Precautions
 Barrier nursing in a side room
 Avoid IM injections (may → infected haematoma)
 Swabs + septic screen
 TPR 4hrly
Antimicrobials
 Start broad spectrum Abx: check local guidelines
 Consider G-CSF

Tumour Lysis Syndrome


Massive cell destruction
 High count leukaemia or bulky lymphoma
↑K, ↑urate, ↑PO4 3- ,↓ Ca → renal failure
 Prevention: ↑ fluid intake + allopurinol

Lymphoma
4 year old boy presented with low grade fever for 2/12 duration, mild pallor and cervical
lymphadenopathy. No organomegaly. Non Hodgkin lymphoma is suspected. What is the best
investigation to diagnose? (INTAKE31)
A. Blood picture
B. CXR
C. Lymph node biopsy
D. Bone marrow biopsy
T

PEDIATRICS MCQ REVIEW – LKSM 12 Page 107


#MMS 33

PEDIATRICS MCQ REVIEW – LKSM 12 Page 108


#MMS 33

HEMATOLOGY

Anemia
Congenital spherocytosis, (COL2021)
a) Is a red cell enzyme defect
b) Causes intravascular haemolysis
c) Presents with neonatal jaundice
d) Is associated with low MCHC
e) Is diagnosed by osmotic fragility test

F F T F T
Microcytic anemia
 Thalassemia
 Iron deficiency
 Anemia of chronic disease
 Sideroblastic
 Pb poisoning

Hereditary Spherocytosis
Commonest inherited haemolytic anaemia in N. Europe
 Pathophysiology
o Autosomal dominant defect in RBC membrane
o Spherocytes get trapped in spleen → extravascular haemolysis
 Features
o Splenomegaly
o Pigment gallstones
o Jaundice
 Complications
o Aplastic crisis
o Megaloblastic crisis
 Ix
o ↑ osmotic fragility
o Spherocytes
o DAT -ve
 Rx
o Folate and splenectomy (after childhood)

Causes for inherited haemolytic anaemia


× Red cell enzyme defects
o G6PD deficiency

PEDIATRICS MCQ REVIEW – LKSM 12 Page 109


#MMS 33

o Pyruvate kinase deficiency


× Membrane defects
o Hereditary spherocytosis
o Hereditary eliptocytosis
× Heamoglobinopathes
o Thalassemia

4 year old girl presented with fever and rash for 2 days.On examination she was febrile and had
erythematous rash on cheeks. She gives a history of neonatal jaundice treated with triple
phototherapy.Her mother had undergone a splenectomy.Investigations :
wbc 2100, (N:78%, L:20),
Hb:4.8,
Plt: 113000,
CRP:12, ESR:5
Direct coombs test- negative
Retic count <1%
What is the most probable diagnosis ?(INTAKE32)
a) Dengue hemorrhagic fever
b) Autoimmune hemolytic Anaemia
c) G6PD deficiency
d) Hereditary spherocytosis with Parvo virus infection
e) SLE

T
Aplastic crisis

Causes for acute sever pallor in a 8 year old boy(INTAKE32)


a. Trauma
b. G6PD
c. Autoimmune haemolytic anemia
d. Parvo B19 infection

T T F T
Oesophageal varices

In hemolytic anemia, (INTAKE31)


A. Bilirubin is present in urine.
B. Pruritus
C. Reticulocyte count more than 2%.
D. Pale stools present.
E. Splenomegaly.
F T T F T

Urobilinogen present in urine

PEDIATRICS MCQ REVIEW – LKSM 12 Page 110


#MMS 33

Pale stools – obstructive jaundice


12-Year-old girl, presented with menorrhagia. On examination she was pale, pulse rate was 120bpm.
There were petechial patches over the anterior abdomen. Rest of the examination is unremarkable.
Investigations are as follows; Hb 4 g/dl, Platelets 10 x 10⁹/L. Next step of management? (COL2021)
a. IV immunoglobulin
b. Urgent gynecology referral
c. Arrange red blood cell transfusion
d. Arrange a test for thyroid function
e. Commence on oral tranexamic acid

Iron deficiency anemia


Features of iron deficiency anaemia, (COL2016)
a) Cognitive impairment is associated

T
There is evidence that iron deficiency anaemia may be detrimental to behaviour and intellectual
function

Best indicator of responding iron treatment in 6 year old girl(INTAKE 30)


1. Increase retic count
2. Increase Hb
3. Increase serum iron
4. Increase ferritin
5. Increase mcv
T

Retic count will increase with 3 dayss


HB rise one in a week
Best – ferritin

Sequence of changes during correction of iron deficiency:

 Clinical improvement in the child (increase in appetite, improvement in irritability)


 Inital bone marrow response
 Increased reticulocyte count
 Haemoglobin levels return to normal
 Body iron stores return to normal (ferritin levels)

Which is NOT true about HbF(INTAKE31)


A. Does not contain B globulin chains
B. Has a higher affinity for O2 than HbA
C. Easily denatured by alkali

PEDIATRICS MCQ REVIEW – LKSM 12 Page 111


#MMS 33

D. Predominant at delivery
E. APT test can be used to differentiate from HbA
T

APT test
The blood is placed in a test tube; sterile water is added to hemolyze the RBCs, yielding free
hemoglobin. This solution then is mixed with 1% sodium hydroxide. If the solution turns yellow-brown,
the hemoglobin is maternal or adult hemoglobin, which is less stable than fetal hemoglobin.

Thalassemia
Beta Thalassemia suspected when child is presented with, (INTAKE 29)
A. Hydrops fetalis
B. Target cells in blood film
C. Pale child with splenomegaly
D. Increased direct bilirubin
E. Child presented with multiple bruises

F T T F F
Hydrops fetalis – alpha thalassemia major (deletion of 4 alpha gene )

Multiple bruising -ITP /leukemia

Splenomegaly is due to extramedullary hemophoesis and RBC destruction increase thalassemia patient
can have mild jaundice increase unconjugated /indirect bilirubin

Clinical presentation beta thalassemia major


 Severe anemia /transfusion dependent from 3-6 moths with jaundice
 Faltering of growth or growth failure

Clotting disorders
Diagnose patient with hemophilia A presented with haemarthrosis following a trauma. Which of the
following is most appropriate for his pain management(INTAKE32)
a. Splinting of limb
b. Keep Ice
c. IV morphine
d. Factor replacement

PEDIATRICS MCQ REVIEW – LKSM 12 Page 112


#MMS 33

A 6day neonate was brought to hospital following bleeding from the umbilical stump after a home
delivery. Was well on Ex.
Ix: Hb – 18, Plt – 220, PT - 60 s (prolonged) APTT – 90 s (prolonged),
Most likely cause is: (INTAKE32)
a. DIC
b. Hemophilia A
c. Hemophilia B
d. Von Willebrand disease
e. Vitamin K deficiency

PEDIATRICS MCQ REVIEW – LKSM 12 Page 113


#MMS 33

DERMATALOGY
(COL2021)
Vesicular eruptions are seen in,
a. Varicella-zoster
b. Measles
c. Molluscum contagiosum
d. Roseola infantum
e. Eczema herpaticum

T F F F T

 Molluscum contagiosum - This is caused by a poxvirus. The lesions are small, skincoloured,
pearly papules with central umbilication
 Measles - Clinical features: fever, cough, runny nose, conjunctivitis, marked malaise, Koplik
spots, maculopapular rash
 Roseola infantum - They classically cause exanthem subitum (also known as roseola infantum),
characterised by a high fever with malaise lasting a few days, followed by a generalised macular
rash, which appears as the fever wanes
 Eczema herpaticum – vesicular lesion develop on eczematous rash

Few definition
 < 5 mm fluid fill blister– vesical
 5 mm fluid fill blister – bullous
 >5 mm solid elevation – nodule
 <5 mm solid elevation – papule
 >2 mm red patch – ecchymosis
 <2 mm red patch – petechia

Vesicular ,bullous , pustular


Viral Varicella- zoster
Herpes simplex
Coxsackie
bacterial Impetigo (crusting )
Boils
Staphylococcal bullous
impetigo
Staphylococcal scaled
skin
Other Erythema multiform ;
St .Jonson syndrome

PEDIATRICS MCQ REVIEW – LKSM 12 Page 114


#MMS 33

A 6 month old baby presented with vesicular rash all over the body, there were signs of excoriation.
What is the treatment option? (INTAKE32)
a. Fusidic acid cream
b. Permethrin
c. Aqueous cream
d. benzylbenzoate
e. 1% hydrocortisone

2 months old baby developed generalized itchy wide spread papules, vesicles on face, chest,trunk,
palms, soles. Mother had similar illness with intense itch. Best treatment is, (INTAKE31)
A. Benzyl benzoate
B. Malathion
C. Sulfar
D. Permethrin
E. Immunoglobulin
T

Scabies
 Very itchy burrows, papules, and vesicles – distribution varies with age.
 Scratching leads to excoriation, secondary eczematous, or urticarial reaction often with
secondary bacterial infection.
 Not only the child but also the whole family will need treatment.
 Treatments
o Permethrin cream (5%) should be applied below the neck to all areas and washed off
after 8–12 hours. In babies, the face and scalp should be included, avoiding the eyes.
o Benzyl benzoate emulsion (25%) applied below the neck only, in diluted form according
to age, and left on for 12 hours, is also effective but smells and has an irritant action.
o Malathion lotion (0.5% aqueous) is another effective preparation applied below the
neck and left on for 12 hours.

Regarding childhood skin disorders (COL2016)


a) target lesions are seen in erythema multiforme
b) palpable rash in buttocks is characteristic of HSP
c) perineal candidiasis rash spares the flexures
d) pearl colour papule with central umbilication in molluscum contangiosum
e) secondary infection in scabies not cause rheumatic fever

T T F T F

Secondary infection (streptococcus ) in scabies is a cause rheumatic fever


.

PEDIATRICS MCQ REVIEW – LKSM 12 Page 115


#MMS 33

Causes of nappy rashes

Common
• Irritant (contact) dermatitis
• Infantile seborrhoeic dermatitis
• Candida infection (The rash is erythematous, includes the skin flexures, and there may be
satellite lesions Treatment is with a topical antifungal agent )
• Atopic eczema

Rare
• Acrodermatitis enteropathica
• Langerhans cell histiocytosis
• Wiskott–Aldrich syndrome

Molluscum contagiosum
• Caused by a poxvirus
• The lesions are small, skincoloured, pearly papules with central umbilication
• They may be single but are usually multiple.
• Lesions are often widespread but tend to disappear spontaneously within a year.
• If necessary, a topical anti-bacterial can be applied to prevent or treat secondary bacterial
infection, and cryotherapy for a few seconds only can be used in older children, away from the
face, to hasten the disappearance of more chronic lesions.

PEDIATRICS MCQ REVIEW – LKSM 12 Page 116


#MMS 33

ENDOCRINE

DKA
7 yr old boy presented with a hx of fever & vomiting for 1 day. Patient is dehydrated,RBS-
843mg/dlBlood gas Ph- 7.12 Appropriate management step is, (INTAKE32)
a. NAHCO3, 0.9% NaCl added Kcl, Insulin infusion
b. NAHCO3, 0.9% NaCl without Kcl, Insulin infusion
c. 0.9%NaCl added KCl, NAHCO3, Insulin infusion
d. 0.9%NaCl added KCl, Insulin infusion

DM
A 14 year old girl presented with polyuria, polydipsia for 2 weeks. RBS is 413 mg/dl, and a diagnosis of
Diabetes mellitus was made. What's the best feature suggestive of type 2 diabetes mellitus? (COL2021)
a. Acanthosis nigricans
b. Age of presentation
c. Associated thyroiditis
d. Family history of diabetes
e. Presence of DKA

T
Diabetic suspicion
 Type 2 diabetes should be suspected if there is a family history, in children from the Indian
subcontinent and in severely obese children with signs of insulin resistance (acanthosis
nigricans – velvety dark skin on the neck or armpits, skin tags or the polycystic ovary phenotype
in teenage girls).
o Acantosis nigricans – is a feature of insulin resistant /PCOS

 Type 1 - There is an association with other autoimmune disorders such as hypothyroidism,


Addison disease, coeliac disease and rheumatoid arthritis in the patient or family history

PEDIATRICS MCQ REVIEW – LKSM 12 Page 117


#MMS 33

Congenital hypothyroidism
A 7 days old new born baby's TSH was checked by a public health midwife and was found to have an
abnormal result. The investigations were reported as TSH 15IU (1.5 to 6) and T4 normal. What's the
next step of management of this baby? (COL2021)
a. Isotope scan
b. Reassure parents and discharge
c. Repeat T4 and TSH in 2 weeks
d. Repeat TSH immediately
e. Start Thyroxine immediately

Biochemical criteria used in the decision to initiate treatment


 If capillary blood TSH concentration on neonatal screening is ≥40 mIU/L, perform TSH and fT4
(thyroid function tests) on venous blood and start treatment. If results can be obtained on the
same day, treatment can be withheld till then.
 If capillary TSH concentration is 20-40 mIU/L, repeat venous blood thyroid function tests
(TFTs). Treatment can be withheld until the venous blood TFT results are available, provided
these will be available on the following day.

Decision to start treatment based on the venous blood TFTs


 If venous fT4 concentration is below the normal range for age, treatment should be started
immediately.
 If venous TSH concentration is >20 mIU/L, treatment should be started even if the fT4
concentration is normal.
 If venous TSH concentration is between 6-20 mIU/L in a well-baby with a fT4 concentration in
the normal range for age,
o Repeat TFT in 2 weeks.
o Perform an ultrasound scan (USS) of the thyroid gland.
o If a small/ectopic thyroid gland is seen with TSH 6-20 mIU/L, then irrespective of
fT4value, thyroxine therapy should be started.
o If the thyroid gland is normal on USS with a TSH 6-20 mIU/L and a normal fT4, repeat
TFT every two weeks till TSH normalizes
 All children with CH should have an USS of the thyroid gland.

Congenital hypothyroidism, (COL2021)


a. Screened in Sri Lanka using the heel prick test
b. Is caused by dyshormonogenesis in the majority
c. Thyroid agenesis requires lifelong treatment
d. Requires immediate commencement of treatment
e. Is more common in Down's syndrome

PEDIATRICS MCQ REVIEW – LKSM 12 Page 118


#MMS 33

T F T T T
Causes for CH
2. Ectopic thyroid 25-50%
3. Thyroid agenesis 20-50%
4. Dyshormonogenesis 4-15%
5. Hypothalamic pituitary dysfunction 1-5%
6. Iodine deficiency 5-10%

 Treatment is lifelong with oral replacement of thyroxine, titrating the dose to maintain
normal growth, TSH and T4 levels
 Early treatment of congenital hypothyroidism is essential to prevent learning difficulties

7 days old baby brought with cord blood TSH -75 mU/ml. clinically no signs of congenital
hypothyroidism .What is the next management? (INTAKE 30)
1. Repeat TSH and T4 in one week
2. repeat TSH now and start thyroxin
3. start thyroxin and do TSH in 3 weeks
4. observe 2 weeks for hypothyroidism featurese)

Cord blood is taken blood grouping /Rh /HB /direct coombs and retic count

PEDIATRICS MCQ REVIEW – LKSM 12 Page 119


#MMS 33

NEUROLOGY

Epilepsy
Breath holding spells are common in childhood. What is the feature which leads to think of analternative
diagnosis? (INTAKE 30)
1. Induce by cry
2. Cyanosis
3. Pallor
4. Increase the tone of the limbs
5. Appears at 2 months of age
T

What is least possible with breath holding attacks? (INTAKE 29)


A. Onset of 6 month of life
B. Triggered by crying
C. Loss of consciousness while crying
D. Spontaneous regression
E. Cyanosis while crying
T

A 1-year-old child developed cyanosis and loss of consciousness following an episode of excessive
crying after a fall. On examination, the child is alert and pink. Neurological and cardiovascular
examination is normal. What is the best step of management? (COL2021)
a. Admit for observation
b. Arrange urgent echocardiogram
c. Check serum Calcium
d. Reassure and discharge
e. Do an EEG

T
Blue breath holding attack Pallid breath holding attack
some toddlers when they are reflex anoxic seizures
upset infants or toddlers
has family history
cries, holds his breath in minor head trauma, cold food, fright or
expiration and goes blue fever
hypoxia
behaviour modification asystole
paroxysmal slow-wave discharge on the EEG

PEDIATRICS MCQ REVIEW – LKSM 12 Page 120


#MMS 33

3 year old child started having afebrile generalized tonic clonic seizure. GP has given rectal Diazepam.
Presented with gargling noise, SOB and continues to have seizures. What is the immediate
management? (INTAKE 29)
A. Suck out secretions and give Oxygen
B. Repeat rectal diazepam
C. IV Midazolam
D. IV Diazepam
E. Saline bolus
T

Acute seizers’ management

Side effects of Sodium valproate? (COL2021)


A. Hirsutism
B. Weight gain
C. Visual field defects
D. Teratogenicity
E. Thrombocytopenia

F F F T T
Side effects of valproate : Anaemia / aggression /deafness /diarrhoea /convulsion /gastric irritation /
haemorrhage /hyponatremia /headache /stupor /thrombocytopenia /transient hair loss /tremor
/nystagmus /nausea

Regarding absence seizures (COL2016)


a) Responds well to carbamazepine
b) Imaging of brain should be advised
c) Event precipitated by hyperventilation
d) Eye lid fluttering is associated during the event

F T T
Neuroimaging in selected cases – MRI better than CT
 All epilepsies under 1 year
 Any suspicion about a SOL
 Uncontrolled epilepsy
 All focal epilepsies (except benign rolandic epilepsy )
 Infantile spasm

Absence seizers
 Transient loss of consciousness
 Abrupt onset and termination
 No motor phenomena
 But has small flickering of eyelids and minor alteration in muscle tone
 Precipitated by hyperventilation

PEDIATRICS MCQ REVIEW – LKSM 12 Page 121


#MMS 33

 Avoid carbamazepine *avoid absence seizures, myoclonic seizures or juvenile myoclonic


epilepsy

Meningitis /encephalitis
A 3 year old boy presented with unsteady gait and slurred speech for one day duration. He had an URTI
with a generalized vesicular rash 3 days ago. His CT is normal. CSF is normal except for mild elevation of
protein. What is the next step of management? (COL2021)
a. Urgent MRI brain
b. IV acyclovir
c. Commence IV antibiotics
d. Commence IV methylprednisolone

T
Cerebellar ataxia causes
 medication and drug
 varicella infection
 posterior fossa lesions or tumours
 genetic and degenerative disorders
o e.g. ataxic cerebral palsy, Friedreich ataxia and ataxia telangiectasia

Encephalitis
 Caused by:
o Direct invasion of the brain by a neurotoxic virus (such as HSV)
o Delayed brain swelling following a dysregulated neuro immunological response to an
antigen, usually a virus (post infectious encephalopathy), e.g. following chickenpox
o A slow virus infection, such as HIV infection or subacute sclerosing pan encephalitis (SSPE)
following measles.
 Most common causes
o Enteroviruses
o Respiratory viruses (influenza viruses)
o Herpesviruses [e.g. HSV, varicella zoster virus (VZV) (HSV is a rare cause of childhood
encephalitis but it can have devastating long-term consequences)
 Onset can be insidious and includes behavioural change
 Treat potential HSV with parenteral high-dose acyclovir (for 3 weeks, as relapses may occur
after shorter courses.) until diagnosis is excluded.

CSF
 Appearance – clear
 WBC –normal / ↑lymphocytes
 Protein – normal /↑
 Glucose – normal /↓

PEDIATRICS MCQ REVIEW – LKSM 12 Page 122


#MMS 33

2-year-old baby, with 4 days’ history of fever, coryzalsynt & convulsions. Treated as for bacterial
meningitis. After 72 hours developed increased fever spikes and drowsiness. What is the likely cause?
(COL2016)
a) Subdural effusion
b) Meningo-encephalitis
c) Cannula site infection
d) SIADH
e) TB meningitis

Post bacterial pneumonia complications


 Collections of fluid in the subdural space develop in 10-30% of patients with meningitis and are
asymptomatic in 85-90% of patients.
 Subdural effusions are especially common in infants.
 Symptomatic subdural effusions may result in a bulging fontanel, diastasis of sutures, enlarging
head circumference, emesis, seizures, fever, and abnormal results of cranial transillumination.
 CT or MRI scanning confirms the presence of a subdural effusion.
 In the presence of increased ICP or a depressed level of consciousness, symptomatic subdural
effusion should be treated by aspiration through the open fontanel
 Fever alone is not an indication for aspiration.
 SIADH occurs in some patients with meningitis, resulting in hyponatremia and reduced serum
osmolality. This may exacerbate cerebral edema or result in hyponatremic seizures
 Fever associated with bacterial meningitis usually resolves within 5-7 days of the onset of
therapy. Prolonged fever ( > 10 days) is noted in about 10% of patients. Prolonged fever is
usually due to intercurrent viral infection, nosocomial or secondary bacterial infection,
thrombophlebitis, or drug reaction

Meningo enchepalitis
o Older children are headache and hyperesthesia
o Infants, irritability and lethargy
o Headache is most often frontal or generalized; adolescents frequently complain of retrobulbar
pain. Fever, nausea and vomiting, photophobia, and pain in the neck, back, and legs are
common.
o As body temperature increases, there may be mental dullness, progressing to stupor in
combination with bizarre movements and convulsions.
o Focal neurologic signs may be stationary, progressive, or fluctuating.

PEDIATRICS MCQ REVIEW – LKSM 12 Page 123


#MMS 33

Febrile convulsions
13 month old baby with a past history of febrile convulsion admitted with fever for 1 day and one
episode of convulsion. During the physical examination, he developed another convulsion. Temperature
is 102 F. Airway, breathing and circulation secured. What is the next best step of management? (COL2021)
a. Give buccal midazolam
b. Administration of paracetamol suppository
c. Buccal diazepam
d. Monitoring vital signs and observe
e. Tepid sponging

Inflammatory polyneuropathy
7-year-old boy presents with ataxia, diplopia, areflexia, and lower limb weakness. One week ago he
had AGE. On examination, there is truncal ataxia and left-sided 6th nerve palsy. What is the best
diagnostic investigation for this condition? (COL2021)
a. CPK levels
b. CSF analysis
c. EEG
d. MRI Brain
e. Nerve conduction studies

 Miller Fisher syndrome the clinical spectrum of GBS extends to an acute motor axonal
neuropathy (AMAN)
 Ocular muscle palsies and ataxia.
 Diagnosis
o Confirmed by nerve conduction studies; these show slowing of conduction in the
common demyelinating form, prolonged distal motor latency and/or conduction block.
o CSF
 protein is often raised
 Cell count and glucose level remain normal.
o Antibodies against GQ1b (ganglioside) have a sensitivity of 90%.

Acute flaccid paralysis (GBS) (INTAKE32)


a. Nerve conduction is reduced in lower leg
b. Affects both proximal and distal muscles
c. Weakness is symmetrical
d. Known to happen following vaccines

PEDIATRICS MCQ REVIEW – LKSM 12 Page 124


#MMS 33

e. Presence of sensory symptoms in the lower limbs rules out the diagnosis

T F T T F

Headache
Features favouring a diagnosis of migraine, (COL2021)
a) Unilateral headache
b) Aggravated by exercise
c) Awakens the child from sleep
d) Associated with photophobia
e) Spontaneous resolution following vomiting

T T F

Triggers
 CHOCOLATE
 CHeese
 OCP
 Caffeine
 alcohOL
 Anxiety
 Travel
 Exercise

Red flag symptoms of headache


 Headache – worse lying down or with coughing and straining
 Headache – wakes up child (different from headache on awakening, not uncommon in
migraine)
 Associated confusion, and/or morning or persistent nausea or vomiting
 Recent change in personality, behaviour or educational performance

Migraine without aura


 90% of migraine
 episodes may last 1–72 h
 headache is
o commonly bilateral but may be unilateral
o Characteristically pulsatile
o over temporal or frontal area
o it is often accompanied by unpleasant gastrointestinal disturbance such as nausea,
vomiting and abdominal pain and photophobia or phonophobia (sensitivity to sounds).

PEDIATRICS MCQ REVIEW – LKSM 12 Page 125


#MMS 33

Migraine with aura


 10% of migraine
 Headache is preceded by an aura (visual, sensory or motor), although the aura may occur
without a headache.
o Negative phenomena, such as hemianopia (loss of half the visual field) or scotoma
(small areas of visual loss)
o Positive phenomena such as fortification spectra(seeing zigzag lines)

Relieve by sleep within few hours


**Spontaneous relive following vomiting NOT resolve

CP
Most likely cause of abnormal hand movements which disappear during sleep with mild developmental
delay and variable tone(INTAKE32)
a. drug induced dystonic reaction
b. Austism spectrum disorder
c. choreoathetoid CP
d. Rett syndrome
e. Rheumatic chorea

Chorea – irregular,non repetitive


Athetosis - slow writhing movements
Dystonia –simultaneous contraction of agonist and antagonist (twisting appearance )

Regarding cerebral palsy, (INTAKE31)


A. Perinatal insults are the commonest cause.
B. Shows hand preference in 6 months.
C. Birth asphyxia is the commonest cause for spastic diplegia.
D. Intratheacal baclofen is effective.
E. Scissoring can be detected in ataxic CP.

T T F T F

Causes for CP
 Perinatal insult 80 %
o Genetic
o Congenital infections (rubella /CMV /toxoplasma/herpis simplex /other ) TORCH
 During birth 10%

PEDIATRICS MCQ REVIEW – LKSM 12 Page 126


#MMS 33

o HIE
 Post-natal 10%
o Meningitis
o seizures /infantile spasms

× periventricular leukomalacia - Hemiplegic and Diplegic


× HIE-Quadriplegic
× Kernicterus- Dyskinetic / choreoathetoid
× Genetic of acquired brain injury- Ataxic

Regarding diplegic cerebral palsy(INTAKE 30)


1. upper limbs are not affected
2. brisk knee jerk
3. mental retardation is common
4. scissoring is seen
5. walking is delayed
F T F T

diplagic – legs > arms


UMN lesion so brisk knee jerk
Intellectual disability common in quadriplegic CP /but diplagic also has
Abnormal walking
Common cause – peri ventricular leukomalacia

MG
A four-year-old child is noticed to hold his eyelids open with fingers specially towards the evening. She
had some difficulty swallowing. But she is able to run and throw a ball well. What is the investigation
you will do to diagnose the condition? (COL2016)
a) ANA
b) Muscle biopsy
c) Edraphonium test
d) Electomyography
e) Creatinine kinase level

T
Edrophonium is not recommended for use in infants; its effect is too brief for objective assessment and
an increased incidence of acute cardiac arrhythmias is reported in infants, especially neonates, with this
drug.

PEDIATRICS MCQ REVIEW – LKSM 12 Page 127


#MMS 33

Intra cranial hypetension


Causes for benign intra cranial hypetension(INTAKE 30)
1. Vitamin A overdose
2. Steroids
3. Sodium valproate
4. Doxycycline
5. Digitalis
T T F T F

Nitrofurantoin /nalidixic (under 6 months) /tetracycline / OCP / steroids /vitamin A /amiodarone


/penicillin /carbidopa /levodopa /cyclosporine / Growth hormone

Causes for open anterior fontanel(INTAKE 30)


1. craniosynostosis
2. subdural effusion
3. interventricular hemorrhage
4. treatment with nalidixic acid
5. rickets
F T T T T

Causes for hydrochepalys

Communicating -
 congenital aqueduct stenosis
 Chari malformation
 dandy walker malformation
 posterior fossa tumor
 intraventricular hemorrhage

Non communicating –
 SAH
 bacterial meningitis
 mumps
 intra uterine infections

PEDIATRICS MCQ REVIEW – LKSM 12 Page 128


#MMS 33

Neural tube defects


Term baby presented with a Meningomyelocele and Hydrocephalus. What is the possible cause? (INTAKE 29)
A. GDM
B. SLE of the mother
C. Folic acid deficiency
D. Prednisolone use by mother for Asthma

T
Neural tube defects
Failure of normal fusion of neural plate to form the neural tube during first 28 days following conception
Incidence is low now due to Natural decline – improved maternal nutrition (low dose folic acid)
× 10 risk for 2nd baby – give high dose folic acid supplementation preconception ally

Meningomyelocele
 Variable paresis of LL with hypotonia
 Muscle imbalance – cause dislocation of hip and talipes
 Sensory loss
 Urinary/bowel incontinence
 Scoliosis
 Arnold Chiari malformation and hydrocephalus

Muscular dystrophies

True regarding Duchenne Muscular Dystrophy, (INTAKE 29)


A. Increased muscle Dystrophin
B. Proximal myopathy
C. Increased CPK
D. Progressive disease
E. Affects intellectual performances

T T T T F
uchenne
 There is absence of the gene product dystrophin
 Average diagnosis at age 5
 Wheel chair bound 10 -15
 Die late 20

Becker’s
 dystrophin levels are present but low
 Slow progression
 Wheel chair bound age 40

PEDIATRICS MCQ REVIEW – LKSM 12 Page 129


#MMS 33

OTHERS

Pathognomic clinical feature


Pathognomic clinical feature is(INTAKE31)
A. Koplic spots in measles
B. Cherry red spots in Taysach syndrome
C. Cafe au lait spots in neurofibromatosis
D. Bitot spots in Vit A deficiency
E. Rose spots in Typhoid
T

Other pathognomic
 Scabies – burrows
 Auer rods- AML
 Aerophobia (fear of air) - rabies.
 Bilateral INO is almost pathognomonic of MS.
 Reed-Sternberg (RS) cell, a pathognomonic feature of HL
 Cateplexy is considered pathognomonic for narcolepsy
 Warthin-Finkeldey giant cells that are pathognomonic for measles

Rose spots - shigellosis and nontyphoidal salmonellosis.

PEDIATRICS MCQ REVIEW – LKSM 12 Page 130


#MMS 33

Least important for diagnosis is, (INTAKE 29)


A. DHF – Positive Hess test
B. Infective endocarditis – Osler nodes
C. Measles – Koplik’s spots
D. Vitamin A deficiency – Bitot's spots
T

 Osler node 5%
 Osler node – SLE
 Bitot’s spots – white plaques of keratinized epithelial cells – are found on the conjunctiva of
young children with vitamin A deficiency. These spots can, however, be seen without vitamin A
deficiency, possibly caused by exposure.
 Capillary fragility and thrombocytopenia – Hess test positive
 DF – endemic area fever with 2 of following
o Positive Hess test
o Nausea
o Vomiting

Drugs
Drug dosage of children are calculated by
1. BW
2. BMI
3. Body surface area – steroids
4. Height
5. Age
T F T F T

What are the factors considered in calculating drug doses for children, (INTAKE 29)
A. Age
B. Body weight
C. Height
D. Body surface area

PEDIATRICS MCQ REVIEW – LKSM 12 Page 131


#MMS 33

E. BMI
T T F T F

Which of the following causes gum hypertrophy as side effect? (COL2016)


a) Sodium valproate
b) Phenytoin sodium
c) Cyclosporine
d) Methotrexate
e) Sulphasalazine

F T T F F
Gingival swelling Inflammation (e.g. pregnancy, scurvy) infiltrations (leukaemia) or fibrous hyperplasia
(e.g. drug-induced – phenytoin, ciclosporin, nifedipine)

Not used as 1st line under the age of 7 years(INTAKE31)


A. Ciprofloxacin
B. Pheylmethyl Penicillin
C. Co amoxyclav
D. Amoxicillin
E. Cefotaxime
T

PEDIATRICS MCQ REVIEW – LKSM 12 Page 132


#MMS 33

STATISTICS
Still births, live births, early neonatal deaths are most useful in calculating(INTAKE 29)
A. Perinatal mortality rate
B. Infant mortality rate
C. Neonatal death rate
D. Postnatal death rate
E. Still birth rate

 Perinatal mortality rate (PNMR) number of stillbirths + number of early neonatal deaths per
1000 total deliveries

 Infant mortality rate is the number of infant (dead before 1st birthday) deaths for every 1,000
live births.

 Neonatal death rate: Number of individuals dying at less than 28 days of age per 1,000 live
births

 Post neonatal mortality rate is the number of resident newborns dying between 28 and 364
days of age in a specified geographic area (country, state, county, etc.) divided by the number of
resident live births for the same geographic area (for a specified time period, usually a calendar
year) and multiplied by 1,000

 Still birth – death after the 24th gestation

Environmental history least important in the diagnosis of(INTAKE31)


A. Typhoid
B. Brachial asthma
C. Rheumatic fever
D. Rheumatoid arthritis
E. Impetigo
T

PEDIATRICS MCQ REVIEW – LKSM 12 Page 133


#MMS 33

GOOD LUCK!
PEDIATRICS MCQ REVIEW – LKSM 12 Page 134

You might also like